Study

¡Supera tus tareas y exámenes ahora con Quizwiz!

A client diagnosed with Celiac disease is on a gluten-free diet. What statement by the client would indicate to the nurse that reinforcing of diet instructions is needed? You answered this question Incorrectly 1. "I will still have occasional abdominal discomfort." 2. "I may need to take iron or vitamin supplements." 3. "I can have eggs but no wheat toast for breakfast." 4. "I should avoid fresh apples and strawberries."

Celiac disease is an immune response that tends to have a familial connection. Remember that the disease causes an inflammation of the intestinal villi, resulting in poor nutrient absorption. When any type of gluten is introduced into the digestive track, the client experiences severe symptoms such as gas and bloating, diarrhea, weight-loss, and chronic fatigue with weakness. Over time, the client may develop anemia, osteoporosis, or even pancreatic problems. So, this is a very serious disease that has no known cure. Clients are taught how to live with this process by following a strictly gluten-free diet. Until recent years, it was difficult to find enough gluten-free foods to plan a nourishing, tasty diet. Gluten free products are now more readily available. Now, examine the stem of the question. The nurse is looking for a statement by the client indicating that reinforcement of diet instruction is needed - so the nurse is looking for an inaccurate comment by the client. When reading each option, ask yourself, is this a truthful statement about Celiac disease? You are looking for what the client said that was incorrect. Option 1: Well this is certainly an accurate statement by the client - no matter how carefully the client with Celiac disease tries to eat gluten-free, there may be hidden gluten not indicated on the packaging in some foods. The inadvertent ingestion of gluten can produce uncomfortable or even painful symptoms such as bloating, gas, or severe headaches. The client acknowledges this might happen, making this an accurate comment by the client. No reinforcement of teaching needed here - try again! Option 2: The scenario states the client has just been diagnosed with Celiac disease, and teaching has been initiated. Because diagnosing this disease can be challenging to a healthcare provider, the client may have developed complications over time such as osteoporosis or anemia. Therefore, it would not be unusual for a client to need to take fat soluble vitamins or iron tablets, among other replacements, to correct any pre-existing complications. This is an accurate statement by the client, indicating the client understands the diet. But you are looking for evidence the client needs more reinforcement of teaching! Try again. Option 3: This statement indicates the client definitely understood the nurse's explanation! Gluten is very prominent in grain products such as wheat, rye, and barley and when ingested, causes extreme pain, bloating, and diarrhea in those with Celiac disease. The appropriate diet for these clients includes dairy and eggs, meat and poultry, seafood, soy and beans, among other foods. The biggest offenders for causing discomfort are wheat, rye, and barley products. This statement definitely indicates the client understands that eggs are acceptable, while avoiding wheat toast. Not the option you are looking for! Option 4: Great choice! It is obvious from this statement by the client that reinforcement of teaching is certainly necessary. While a gluten-free diet does have many foods to be avoided, fresh fruits and vegetables are not only fitting but encouraged for their vitamin content. Fresh apples and strawberries are a great addition for client with Celiac disease because of poor nutrient absorption. This client need to be cautious about canned fruits or veggies, because sometimes malt barley (high in gluten) is added as a thickening agent - that would cause a problem! But fresh fruits and veggies are a great idea! This statement indicates the client needs reinforcement of teaching on the basics of the gluten-free diet.

In what order, after initially washing hands, should the nurse change a dressing on an infected abdominal surgical wound that has a Penrose drain and a large amount of purulent drainage? Place in priority order from first to last.

First, apply clean gloves. Second, remove soiled dressings. Third, discard soiled dressings and clean gloves in red bag. Fourth, don sterile gloves. Fifth, clean surgical wound with moistened sterile 4x4's. Sixth, clean around Penrose drain using circular pattern inside to outside. Seventh, place dry, sterile 4x4's over surgical wound and Penrose drain. Eighth, apply abdominal dressing pad.

Immediately after a liver biopsy, a client is placed on the right side for 60-90 minutes. What is the rationale for placing the client in this position? You answered this question Correctly 1. Helps stop bleeding if any occurs. 2. Restores circulating blood volume. 3. This is the position of greatest comfort. 4. Helps reduce fluid trapped in the biliary ducts.

1. Correct: Anyone who has a liver biopsy is at risk for bleeding. The clotting factors are produced in the liver, as is prothrombin. Any time a needle is inserted into the body and removed, bleeding can occur. Whenever there is a risk for bleeding, the preventive measure is to apply pressure. Lying on the right side applies pressure to the liver. A towel may be rolled up and placed under the right side for added pressure. Again, pick the most life threatening answer. This is what could kill the client. 2. Incorrect: Lying on the right side does not lead to restoration of circulating blood volume. 3. Incorrect: Lying on the right side is not a position of comfort. Applying pressure is the goal, as the client is at risk for bleeding. 4. Incorrect: Lying on the right side does not reduce fluid trapped in the biliary ducts.

A client who presents with severe epigastric pain, reports that three rolls of calcium carbonate were consumed in the past eight hours to treat the indigestion. Which blood gas report does the nurse associate with this situation? You answered this question Correctly 1. pH - 7.49, pCO2 - 40, HCO3 - 30 2. pH - 7.32, pCO2 - 48, HCO3 - 20 3. pH - 7.38, pCO2 - 52, HCO3 - 32 4. pH - 7.29, pCO2 - 54, HCO3 - 26

1. Correct: These ABGs are indicative of metabolic alkalosis. The pH is high, the pCO2 is within normal limits and the bicarb is high (alkalosis). So, the excess Tums (calcium carbonate) could have caused metabolic alkalosis. 2. Incorrect: The client is not hypoventilating and would not be in metabolic acidosis because he ate 3 rolls of Tums, which is a base. These ABGs are indicative of acidosis. The pH is low (acidosis), the pCO2 is high (acidosis) and the bicarb is low (acidosis). 3. Incorrect: The client is not a long-term COPD client as these ABGs might suggest. These ABGs are indicative of fully compensated respiratory acidosis. The pH is normal. The pCO2 is high (as with chronic retention) and the bicarb is high to help compensate. 4. Incorrect: These ABGs are the result of an acute ventilation problem. They are indicative of respiratory acidosis. The pH is low, the pCO​2 is high, and the bicarb is normal. No compensation has begun at this point.

Which nursing intervention should receive priority after a client has returned from having had eye surgery? You answered this question Incorrectly 1. Administer pain medication around the clock. 2. Maintain head of bed at 35°. 3. Alternate applying warm and cold compresses. 4. Instruct on importance of turning, coughing, and deep breathing.

2. Correct: Maintaining head of bed in an elevated position will help to decrease intraocular pressure. Do not lie the client supine as this will increase intraocular pressure. If the intraocular pressure increases too much, damage to the eye structures, including the retina and optic nerve, with resulting loss of vision may result. 1. Incorrect: Although pain management is important, it is not the priority here. The priority intervention of maintaining the bed at 35° is to reduce the risk of increased intraocular pressure. Unless the pain becomes out of proportion or suddenly worsens, it is an expected finding and would not be cause for alarm or require "priority" attention. 3. Incorrect: Warm and hot compresses could possibly increase intraocular pressure and cause damage to the eye structures. 4. Incorrect: Coughing will increase intraocular pressure and could result in damage to the surgical site and/or the structure within the eye. Loss of vision could result if pressure becomes too great. Coughing is a type of valsalva movement which results in an increase in the intraocular pressure.

Which nursing intervention can the LPN/LVN safely perform? You answered this question Correctly 1. Assess a client for a hearing loss. 2. Reinforce hand-washing with the client who has bacterial conjunctivitis. 3. Evaluate a client's ability to instill eye medication. 4. Create the plan of care for a client post cataract surgery.

2. Correct: The LPN/VN can reinforce education. The LPN/VN must know the scope of practice of the LPN/VN 1. Incorrect: The LPN/VN cannot instruct, assess, evaluate, or create the plan of care. These are RN tasks only. 3. Incorrect: The LPN/VN cannot instruct, assess, evaluate, or create the plan of care. These are RN tasks only. 4. Incorrect: The LPN/VN cannot instruct, assess, evaluate, or create the plan of care. These are RN tasks only.

What nursing interventions should the nurse implement for a client with Addison's disease? You answered this question Incorrectly 1. Administer potassium supplements as prescribed. 2. Assist the client to select foods high in sodium. 3. Administer Fludrocortisone as prescribed. 4. Monitor intake and output. 5. Record daily weight.

2., 3., 4. & 5. Correct: The client with Addison's disease needs sodium due to low levels of aldosterone. Fludrocortisone is a mineralocorticoid that the client will need to take for life. I&O and daily weights are needed to monitor fluid status. 1. Incorrect: Clients with Addison's disease lose sodium and retain potassium, so this client does not need potassium.

The nurse is caring for a client on the psychiatric unit. The client is prescribed fluphenazine 10 mg. The drug is available as an elixir: 2.5 mg / 5 mL. How many mL will the nurse give to the client? Round answer to the nearest whole number. You answered this question Correctly Enter the answer for the question below.

2.5 mg : 5 mL = 10 mg : x mL 2.5 mg/x mL = 50 mg/mL 2.5 mg/x mL = 50 mg/mL x = 20 mL

A nurse has reinforced teaching to a client about home dressing changes using a clean technique. Which statement made by a client indicates to the nurse that the client understands this technique? You answered this question Correctly 1. "The wound should be cleaned using a washcloth, soap, and water." 2. "Povidone-iodine should be applied to the wound with each dressing change." 3. "It is important that I wash my hands using soap and water before removing my dressing." 4. "I will use sterile gloves to clean my wound and change the dressings."

3. Correct: Clean technique requires washing hands with soap and water prior to removing the dressing. 1. Incorrect: The wound should be cleaned with 4x4's and sterile water. Soap can be very drying to the wound. A washcloth may not be clean as it has been sitting in a cabinet. 2. Incorrect: Povidone-iodine is harsh and damages healthy tissue, so should not be applied to the wound. 4. Incorrect: Sterile gloves are not needed when using clean technique. Clean gloves may be used.

Which nursing action is likely to improve client satisfaction and demonstrate acts of beneficence? You answered this question Correctly 1. Allowing clients to make their own decisions about care 2. Answering all questions posed by client in an honest manner 3. Reporting faulty equipment to the proper departments 4. Sitting at the bedside and listening to an elderly client

4. Correct: Sitting and listening demonstrates kindness and compassion that are consistent with the ethical term "beneficence." Beneficence is taking positive action to help others and desiring to do good which are the core principles of client advocacy. 1. Incorrect: Autonomy is the ethical principle illustrated here by supporting independent decision making with clients. 2. Incorrect: Fidelity is the ethical principle illustrated here and refers to the concept of keeping a commitment. It is based upon the virtue of caring. 3. Incorrect: Reporting faulty equipment is an act to promote nonmaleficence or to do no harm. This is the core of nursing ethics.

A client diagnosed with arachnophobia is prescribed alprazolam 0.5 mg orally three times daily. The nurse knows that reinforcement of teaching about this medication is successful when the client makes what statement? You answered this question Correctly 1. Alprazolam will take up to two weeks to start working. 2. The drug does not cause drowsiness, so my daily activities will not suffer. 3. This medication cannot be taken with food. 4. I should not stop taking alprazolam suddenly.

4. Correct: Suddenly stopping alprazolam could produce serious withdrawal symptoms, such as depression, insomnia, anxiety, abdominal and muscle cramps, tremors, vomiting, sweating, convulsions, and delirium. 1. Incorrect: Alprazolam works relatively quickly. 2. Incorrect: Drowsiness, confusion, and lethargy are the most common side effects. The client should not drive or operate dangerous machinery while taking the medication. 3. Incorrect: If the client experiences nausea and vomiting, take with food or milk.

The primary healthcare provider prescribes nafcillin 0.6 gram every 12 hours IM. Available is a vial labeled 200 mg per 1 mL. How many mL should the nurse give? Round your answer to the nearest whole number. You answered this question Correctly Enter the answer for the question below.

Changing 0.6 g to mg equals 600 mg. Then 200 mg : 1 mL = 600 mg : x mL 200x = 600 x = 3

The nurse is caring for a client taking a selective serotonin reuptake inhibitor (SSRI). The client tells the nurse "I am sweating more than ever!" What is the nurse's best response? You answered this question Incorrectly 1. This is a common side effect of antidepressant medications. 2. Excessive sweating can have many causes. 3. You should report this side effect to your primary healthcare provider. 4. This symptom should go away within a few days.

1. Correct: A common side effect of SSRIs is increased sweating. This option gives the client an explanation. 2. Incorrect: This response shows a lack of understanding of the side effects of antidepressant medications. 3. Incorrect: This option does not acknowledge the client's problem and possible causes. 4. Incorrect: Increased sweating may continue throughout treatment with an antidepressant medication.

A client consumes a lacto-ovo vegetarian diet at home. During hospitalization, the primary healthcare provider prescribes an increased calorie diet. Which foods are appropriate for the nurse to serve as between meal snacks to boost caloric intake? You answered this question Incorrectly 1. Cheese sandwich and milk 2. Boiled eggs but no dairy products 3. Fish sticks and cocktail sausages 4. Fresh vegetables but no milk or eggs

1. Correct: A lacto-ovo vegetarian diet is a vegetarian diet that does not include meat, but does contain eggs and dairy. The client can eat milk and dairy products along with grain products on this diet. 2. Incorrect: Dairy products and eggs are allowed on this diet. Milk, cheese and yogurt can be consumed on a lacto-ovo vegetarian diet. 3. Incorrect: The client does not consume meats. Meats, fish, and poultry should not be provided as a snack. 4. Incorrect: The client can consume milk and eggs as well as fresh fruits and vegetables. Milk and eggs can be consumed on a lacto-ovo vegetarian diet.

A nurse is caring for a client hospitalized with Guillain-Barre syndrome. Which is the most important nursing measure for this client? You answered this question Correctly 1. Observation and support of ventilation 2. Insertion of indwelling urinary catheter 3. Nasogastric suctioning 4. Frequent monitoring of level of consciousness

1. Correct: Guillain-Barre syndrome is an acquired inflammatory disease that results in demyelinization of the peripheral nerves. It is usually ascending in nature and can lead to respiratory paresis or paralysis. 2. Incorrect: Insertion of an indwelling urinary catheter may in fact be necessary but does not prioritize higher than support of ventilation. 3. Incorrect: Nasogastric suctioning is not a need identified with Guillain-Barre syndrome. 4. Incorrect: The client's cognitive function remains intact, and there is no data in the stem of the question that indicates otherwise; therefore, ventilation is the priority. Guillain-Barre does not affect the LOC.

Which comment by the client indicates understanding of possible complications of long term hypertension? You answered this question Correctly 1. "I would like to have my serum creatinine checked at this visit." 2. "My blurred vision is part of getting older." 3. "I have leg pain caused by excessive exercise." 4. "Adding salt to my food is permissible."

1. Correct: Hypertension is one of the leading causes of end stage renal disease. The client understands that renal function is reflected by serum creatinine levels. This request demonstrates understanding of the disease and possible complications. 2. Incorrect: The appearance of the retina provides important information about the severity and duration of hypertension. Manifestations of severe retinal damage include blurred vision, retinal hemorrhage, and loss of vision. 3. Incorrect: Intermittent claudication is a complication of peripheral vascular disease (PVD). Hypertension speeds up the process of PVD. 4. Incorrect: Increasing the salt intake causes an increase in the sodium levels which can reduce the kidney's ability to remove excess fluid. This can result in a worsening of the hypertension. Lifestyle modifications to reduce hypertension include dietary sodium reduction, weight reduction, Dietary Approaches to Stop Hypertension (DASH) eating plan, moderation of alcohol consumption, regular physical activity, avoidance of tobacco use, and management of psychosocial risk factors.

A client in the inpatient mental health unit has been determined not suicidal. The client is requesting to leave against medical advice (AMA). What should the nurse do first? You answered this question Incorrectly 1. Inform the primary healthcare provider that the client is wishing to leave. 2. Make arrangements for a commitment hearing 3. Tell her that she must stay until her primary healthcare provider discharges her. 4. Call the primary healthcare provider and request a discharge order.

1. Correct: Protocols on the unit must be followed when someone is requesting to leave AMA. The first step involves calling the primary healthcare provider. 2. Incorrect: The client is not suicidal; however, the primary healthcare provider needs input into this decision. 3. Incorrect: If the client is not a threat or potential threat to self or others, the client may leave. But first the nurse must inform the primary healthcare provider. 4. Incorrect: The nurse should call the primary healthcare provider and discuss the situation. The primary healthcare provider should have input into this decision.

A client has been on the medical unit for three days and is requesting to leave against medical advice (AMA). It has been determined that the client is not suicidal. What should the nurse do? You answered this question Correctly 1. Inform the primary healthcare provider that the client wishes to leave. 2. Make arrangements for a commitment hearing as soon as possible. 3. Tell the client the primary healthcare provider must discharge the client prior to leaving. 4. Call the primary healthcare provider and request a discharge order.

1. Correct: Protocols on the unit must be followed when someone is requesting to leave AMA. The first step is to call the primary healthcare provider about the client's desire to leave AMA. 2. Incorrect: The client is not suicidal; however, the primary healthcare provider needs input into their decision to leave AMA. It is not appropriate to prepare for a commitment hearing. 3. Incorrect: If the client is not a threat or potential threat to self or others, the client may leave. The nurse may discuss the decision to leave; however, this statement is not accurate. 4. Incorrect: The nurse should call the primary healthcare provider and discuss the situation. The primary healthcare provider should have input into this decision.

A client presents to the after-hours clinic with reports of pain that occurs with walking but generally subsides with rest. The nurse notes coolness and decreased pulses in lower extremities bilaterally. What condition would the nurse recognize these symptoms being most indicative of? You answered this question Incorrectly 1. Chronic Arterial Insufficiency 2. Chronic Venous Insufficiency 3. Chronic Unstable Angina 4. Chronic Coronary Artery Disease

1. Correct: These symptoms are indicative of arterial insufficiency as there is pain with walking that is relieved by rest. This pain is known as intermittent claudication. In addition, the pulses are decreased or may be absent with arterial insufficiency, and the extremities are cool to touch. Other s/s include: paleness of extremity when elevated or possible redness when lowered, loss of hair on affected extremity, and thick nails. 2. Incorrect: Venous insufficiency is not characterized by pain with walking. Pulses are generally normal and color is generally normal with the exception of the brown pigmentation that may be noted (especially around the ankles). 3. Incorrect: The description in the stem is evident of peripheral arterial insufficiency and is not descriptive of decreased coronary artery perfusion. No reports of chest pain were noted. 4. Incorrect: The description is evident of peripheral arterial insufficiency and is not descriptive of decreased coronary artery perfusion. The symptoms listed in the stem are indicative of a peripheral artery problem.

A psychiatric nurse, caring for several clients, recognizes that which client presents the greatest risk for violence toward others? You answered this question Correctly 1. Twenty four year old man with paranoid delusions. 2. Sixty two year old woman with bi-polar disorder 3. Seventy year old man with major depression. 4. Twenty eight year old woman with borderline personality disorder

1. Correct: This client has a diagnosis that is consistent with a risk of violence, and his age falls within the age range for males who are most likely to present a risk of violence toward others. 2. Incorrect: This client may be irritable; however, it is not likely that she will present a great risk for violence. Her age does not fall within the range for women that are most likely to present a threat of violence. 3. Incorrect: This client is more likely to hurt himself. 4. Incorrect: This client is more likely to present a risk of violence toward self.

Which task would be appropriate for the nurse to assign to an unlicensed assistive personnel (UAP)? You answered this question Incorrectly 1. Totaling I & O records on five clients at the end of the shift. 2. Assessing VS on a client who was admitted 30 minutes ago. 3. Administering nasogastric (NG) tube feeding. 4. Changing an abdominal surgical dressing on a client that is 3 days post op.

1. Correct: Totaling I & O is an appropriate task for a UAP to be assigned. This is within the scope of practice for the UAP. 2. Incorrect: New clients should be assessed by an RN; however, it is acceptable for the RN to get assistance with some of the information. The RN must verify all information. The client is a new admit, and is considered unstable; therefore, the RN should get the baseline vitals. 3. Incorrect: Administering a NG tube feeding is not within scope of practice for the UAP. 4. Incorrect: Changing a surgical dressing is not within the scope of practice for the UAP.

A client who has schizophrenia tells the nurse, "I am Jesus and I am here to save the world". The client is warning others of hell and damnation. The whole unit is getting upset and several are beginning to cry. What action should the nurse take? You answered this question Correctly 1. Set limits and send the client to room. 2. Explain to the client that not all people are Christians. 3. Remove the Bible from the client and explain that the client is not Jesus. 4. Ask the client, "Share with the group how you know that you are Jesus."

1. Correct: Yes! You must set limits of where the client is allowed to preach. This is disrupting others, and the client needs to be redirected to the client's room for a cool down and then another activity shortly thereafter. 2. Incorrect: No, you should not tell the client that not all people are Christians, because the client will then go on a quest of salvation. 3. Incorrect: No, don't argue with the client. 4. Incorrect: This is ridiculing the client and also inflaming the situation

A primipara at 36 weeks gestation is seen in the OB/GYN clinic. Which sign/symptom should the nurse immediately report to the primary healthcare provider? You answered this question Incorrectly 1. Puffy hands and face 2. Reports indigestion 3. Pedal edema 4. Backache 5. Severe headaches rated 9/10

1., & 5. Correct: Facial and upper extremity edema can be a sign of pre-eclampsia, which can endanger both the mother and fetus. Preeclampsia is a pregnancy complication characterized by high blood pressure and signs of damage to another organ system, often the kidneys. Preeclampsia usually begins after 20 weeks of pregnancy in a woman whose blood pressure had been normal. Even a slight rise in blood pressure may be a sign of preeclampsia. Left untreated, preeclampsia can lead to serious, even fatal, complications. Signs and symptoms of preeclampsia include hypertension and may include proteinuria, severe headaches. changes in vision, upper abdominal pain, nausea or vomiting, decreased urine output, thrombocytopenia, impaired liver function, shortness of breath, sudden weight gain, and edema (particularly in face and hands). 2. Incorrect: Indigestion should be assessed for severity, but it is a common symptom in the 3rd trimester of pregnancy. 3. Incorrect: Pedal edema should be assessed but is common in the 3rd trimester of pregnancy. 4. Incorrect: Backache is common in the 3rd trimester. It is caused by the center of gravity shifting.

What risk factors should the nurse identify when screening individuals for type 2 diabetes mellitus? You answered this question Incorrectly 1. Fat distribution greater in abdomen than in hips. 2. Being underweight. 3. Having type 1 diabetes as a child increases risk for type 2 diabetes. 4. Caucasians are more likely to develop type 2 diabetes than Hispanics. 5. Polycystic ovary syndrome.

1., & 5. Correct: If the body stores fat primarily in the abdomen, the risk of developing type 2 diabetes is greater than if the body stores fat elsewhere, such as the hips and thighs. Women with polycystic ovary syndrome have increased risk of developing type 2 diabetes. 2. Incorrect: Being overweight is a primary risk factor for type 2 diabetes. The more fatty tissue, the more resistant cells become to insulin. 3. Incorrect: A type 1 diabetic will remain a type 1 diabetic. 4. Incorrect: Blacks, Hispanics, American Indians, and Asian Americans are more likely to develop type 2 diabetes than Caucasians.

A nurse who has never had varicella has been exposed to a client diagnosed with herpes zoster. What actions should the nurse take? You answered this question Incorrectly 1. Notify the infection control nurse. 2. Continue to care for client as varicella and herpes zoster are not related. 3. Go to the lab to have a Tzanck smear performed. 4. Obtain herpes zoster vaccine for protection from this exposure. 5. Receive the varicella-zoster immune globulin within 96 hours of exposure.

1., & 5. Correct: Notify the person responsible for infection control to get post-exposure treatment initiated within a timely manner. For persons who are susceptible, the varicella-zoster immune globulin should be given within 96 hours of exposure. The infection of herpes zoster is contagious until the crusts have dried and fallen off the skin. 2. Incorrect: Varicella is chickenpox and herpes zoster is shingles. Both are closely related. Exposure to herpes zoster by someone who has not had varicella places the person at risk for developing herpes zoster. 3. Incorrect: A Tzanck test consists of examining tissue from the lower surface of a lesion in a vesicular condition to determine cell type. The Tzanck test is not associated with immunity from the varicella-zoster virus. 4. Incorrect: The vaccine will not prevent the nurse from developing shingles from this exposure. The nurse needs immune globulin for immediate protection.

The nurse determines that a client does not have an advance directive. The daughter is designated to make healthcare decisions in the event that the client becomes incapacitated or unable to make informed decisions. Which nursing actions are appropriate for this client? You answered this question Incorrectly 1. Document the client's statement in the client's own words. 2. Provide information on advance directives to the client. 3. Provide personnel for assistance with completing an advance directive. 4. Encourage client to complete advance directive as soon as possible. 5. Determine if the client's daughter agrees with the client's decision.

1., 2. & 3. Correct: The nurse should document the client's statement in the client's own words. The nurse should provide the client with information on advance directives and assurance that there are hospital personnel to assist with completing the advance directive. 4. Incorrect: The nurse should explain to the client that the law requires all clients be asked about the existence of an advance directive at the time of hospital admission. Preparing an advance directive ensures that the client's wishes will be followed in the event that the client is unable to make healthcare decisions. The decision about an advance directive is the client's decision to make and not the nurse's decision. 5. Incorrect: Providing information is the appropriate nursing action, not questioning the daughter.

When caring for a client on bedrest, which interventions should the nurse implement to decrease the risk of deep vein thrombosis? You answered this question Incorrectly 1. Apply compression hose. 2. Place pillow under knees while supine. 3. Assist client to perform active foot and leg exercises. 4. Place client on intermittent pneumatic compression device. 5. Assess extremities for negative Homan's sign.

1., 3., & 4. Correct. The client will need compression or compression hose and/or intermittent pneumatic compression device. The client should perform leg and foot exercises to decrease stagnation of blood. Compression hose, foot and leg exercises, as well as pneumatic compression devices increase venous return and prevents stasis of blood. Other interventions to decrease deep vein thrombosis (DVT) include early ambulation, passive and active range of motion, isometric exercises, and anticoagulant drugs such as heparin. 2. Incorrect: Do not compromise blood flow by placing pillows under the knees, crossing legs, or sitting for long periods of time. When pillows are left under the knees for an extended time, venous return could be compromised. A pillow under the knees is not a recommended intervention for DVT prevention. 5. Incorrect: Do not assess Homan's sign, as it may dislodge a clot. Homan's sign is not a preventative intervention. Assessing a Homan's sign is considered to be controversial, and this test may contribute to the release or dislodgement of a clot.

The nurse should reinforce which instructions given to the unlicensed assistive personnel (UAP) about care needed to reduce the risk of infection when a client has an indwelling catheter? You answered this question Correctly 1. Check catheter for kinks in the tubing when the client is in the bed or chair. 2. Disconnect the catheter from the bag when measuring output. 3. Wash hands before providing personal care to the client. 4. Ensure that catheter remains secured to the thigh. 5. Make sure that the drainage bag is always below the level of the bladder.

1., 3., 4. & 5. Correct: Tubing that becomes obstructed cannot allow adequate urine flow. The urine flow occurs by gravity. Adequate handwashing before providing care is one defense against infection. Tension on the tubing may cause irritation and subsequent infection. The bag should be below the level of the bladder so that urine flows appropriately. 2. Incorrect: A closed drainage system should be maintained to prevent entry of microorganisms. Disconnecting the catheter from the bag would be incorrect and potentially cause harm to the client.

A nurse is caring for a client diagnosed with Alzheimer's disease. What actions should the nurse initiate? You answered this question Correctly 1. Monitor client's ability to perform activities of daily living. 2. Perform activities of daily living for the client. 3. Place a clock and calendar in client's room. 4. Encourage family to visit client often. 5. Have nursing staff spend time talking and listening to client.

1., 3., 4., & 5. Correct: All of these should be included in this client's care. Monitor the client's ability to perform activities of daily living and allow client to perform alone if capable. Facilitate orientation by placing items such as a clock, newspaper, and calendar. Encourage family to visit to maintain socialization. Plan for staff to spend some time talking and listening to the client. 2. Incorrect: The staff should facilitate client's independence in all activities for as long as they are able. Encouraging self-care and allowing the client to be involved in activity of daily living (ADL) is a basic right and core principle of client-centered care

What information should the nurse reinforce about decreasing the risk of spreading influenza? You answered this question Correctly 1. Influenza is transmitted via the influenza vaccine. 2. Use a shirtsleeve when coughing or sneezing if tissue is not available. 3. Tissues are not effective in decreasing the spread of influenza. 4. Antibiotics are effective in treating influenza.

2. Correct: A shirtsleeve should be used as a barrier when coughing or sneezing if tissue is not available. This prevents germs being spread via the hands. 1. Incorrect: The vaccine contains a dead virus that is not capable of causing influenza. Clients may experience influenza symptoms from the vaccine, but they won't contract the full-fledged virus. 3. Incorrect: Tissues are effective in decreasing the spread of influenza if disposed of in the trash after use. 4. Incorrect: Antibiotics are not effective in treating influenza. Influenza is treated with antipyretics, fluids, and rest.

The nurse is participating in educating a group of parents about the importance of immunizing their daughters against the human papillomavirus (HPV) in an effort to prevent the development of which cancer? You answered this question Correctly 1. Breast 2. Cervical 3. Ovarian 4. Uterine

2. Correct: A vaccine that offers protection from the virus responsible for most cases of cervical cancer is the latest addition to the official childhood immunization schedule. The HPV vaccine is recommended for boys and girls at age 11 or 12 so they are protected before ever being exposed to the virus. 1. Incorrect: This vaccine does not offer protection for breast cancer. HPV does not increase the risk for breast cancer. 3. Incorrect: This vaccine does not offer protection for ovarian cancer. HPV does not increase the risk for ovarian cancer. 4. Incorrect: This vaccine does not offer protection for uterine cancer. HPV does not increase the risk for uterine cancer.

When reinforcing teaching, what symptom would the nurse include as being the most common initial visual change associated with glaucoma? You answered this question Correctly 1. Central vision is lost. 2. Progressive tunnel vision occurs. 3. Sudden flashes of light in the eyes. 4. Eye floaters begin to be noticed.

2. Correct: If glaucoma is not diagnosed and treated early, an individual starts to lose peripheral vision, or the area of vision outside the central field of sight. People who have glaucoma experience tunnel vision and cannot see objects to the side, near the head, or by their feet. Central vision can be lost later if the disease progresses. 1. Incorrect: Central vision loss is the classic visual disturbance for macular degeneration but peripheral vision is usually maintained. 3. Incorrect: Individuals experiencing retinal detachment may have sudden flashes of light in the affected eye, but this is not an initial visual change related to glaucoma. 4. Incorrect: Eye floaters are more common in eye disorders such as retinal detachment or may occur associated with the aging process.

Which baseline data would tell the nurse that a school aged child is at risk for obesity? You answered this question Correctly 1. Spends one hour playing sports or swimming daily. 2. Spends at least two hours watching TV after dinner each day. 3. Assists mom in preparing low carb snacks for the family. 4. Participates in the marching band at school.

2. Correct: Sedentary activities, such as watching television, playing video games, and using a computer to surf the internet or engage with friends can also contribute to obesity and cardiovascular health problems in later life. 1. Incorrect: The more active the child is, the less likely the child is to be overweight. Activity for at least one or more hours per day should be encouraged. 3. Incorrect: Children who are exposed to healthy snacks are less likely to be overweight and are more likely to choose healthy snacks. 4. Incorrect: The marching band is an excellent source of exercise for the child. This information does not support the risk for obesity.

The nurse is caring for a client taking benazepril. Which symptoms would be important for the nurse to report to the primary healthcare provider? You answered this question Incorrectly 1. BP 150/108 decreases to 138/86 2. Weight gain of 5 pounds (2.27 kg) in one week 3. Urinary output of 1450 mL in 12 hours 4. Apical pulse of 90/min

2. Correct: Weight gain of 5 pounds in one week is a s/s of an adverse effect of ACE inhibitor use. Weight gain is a sign of fluid retention. 1. Incorrect: This is an expected response of an ACE inhibitor. ACE inhibitors block the normal effects of renin-angiotensin-aldosterone system, thereby decreasing the blood pressure. 3. Incorrect: The urinary output is adequate and indicates good renal function and perfusion to the kidneys. 4. Incorrect: The apical pulse is normal.

Prior to administering medications, the nurse must identify the client using which identifiers? You answered this question Correctly 1. Room number 2. Date of birth 3. Identification band 4. Client correctly states name 5. Visitor stating client's name

2., 3. & 4. Correct: The client's date of birth and the client's identification band can be used as the two identifiers per Joint Commission standards. The client may also state their name. Two identifiers must be used. 1. Incorrect: The client's room number or visitor statement is not considered a client identifier. 5. Incorrect: The client's room number or visitor statement is not considered a client identifier.

What should the nurse include when reinforcing teaching to a female client prescribed doxycycline for the treatment of acne? You answered this question Correctly 1. Take this medication with food to maximize absorption. 2. Use a non-hormone method of birth control while taking this medication. 3. Wear protective clothing when outside. 4. Drink plenty of fluids while taking this medication. 5. Iron and calcium supplements can be taken with this medication.

2., 3., & 4. Correct: Doxycycline is a tetracycline antibiotic. Doxycycline can make birth control pills less effective. A non-hormone method of birth control (such as a condom, diaphragm, and/or spermicide) should be used to prevent pregnancy while using doxycycline. Avoid exposure to sunlight or tanning beds. Doxycycline can make you sunburn more easily. Wear protective clothing and use sunscreen (SPF 30 or higher) when outdoors. Take doxycycline with a full glass of water. Drink plenty of liquids while taking this medicine. 1. Incorrect: Take on an empty stomach to maximize absorption, although it may not be tolerated unless administered with food. 5. Incorrect: Do not take iron supplements, multivitamins, calcium supplements, antacids, or laxatives within 2 hours before or after taking doxycycline. Absorption will be altered. For instance, iron can bind to doxycycline in the gastrointestinal tract, which may prevent their absorption into the bloodstream and possibly reduce their effectiveness. To avoid or minimize the interaction, iron containing medications and doxycycline should preferably be taken at least three hours apart in most cases.

The nurse is reinforcing teaching to a client who has been prescribed fluticasone/salmeterol. What points are important for the client to understand? You answered this question Correctly 1. Swallow the capsule when having an acute asthma episode. 2. Rinse mouth after medication administration to decrease infection. 3. Take this medication every day, even on days when breathing fine. 4. Administer by inhalation device twice daily. 5. Carry a rescue inhaler, such as albuterol, when leaving home.

2., 3., 4., & 5. Correct: This medication contains a steroid which can increase the risk of oropharyngeal fungal infections. Rinsing will reduce this risk of infection and will also decrease mouth and throat irritation. The medication should be taken every day as directed, even on days when the client feels they are breathing better. This is a preventative medication, not a rescue medication. This medication is administered by an inhaler. It is not given orally. A rescue inhaler, such as albuterol, is needed when the client leaves home. Fluticasone/salmeterol is not a rescue inhaler, but is for long term control and maintenance treatment for the prevention of bronchospasm and airway inflammation associated with asthma, chronic bronchitis, and COPD. 1. Incorrect: This medication must be taken with an inhaler.

Which tasks would be appropriate for the LPN/LVN to assign to an unlicensed assistive personnel (UAP)? You answered this question Incorrectly 1. Monitor client for signs of skin breakdown. 2. Take client's vital signs after ambulating. 3. Apply bacitracin ointment to right forearm. 4. Obtain a stool specimen. 5. Determine what activities the client can do independently.

2., 4. Correct. These tasks are within the scope of practice for the UAP. The LPN/VN must know the tasks that are appropriate for the UAP in order to assign tasks. 1. Incorrect. The UAP cannot assess, evaluate, or plan care for the client. The LPN/VN knows that checking for signs of skin breakdown requires data collection through monitoring. 3. Incorrect. The UAP cannot administer medication. Bacitracin ointment is a medication. 5. Incorrect. The UAP cannot assess, evaluate, or plan care for the client. This task involves data collection and evaluation.

Which task would be appropriate for the LPN/VN to complete? You answered this question Correctly 1. Assessing a client who was just admitted to the unit. 2. Administering morphine IV push to a two day post-op client. 3. Feeding a client through a percutaneous endoscopic gastrostomy (PEG). 4. Reinserting a PICC line that was pulled out by the client.

3. Correct: Feeding by way of a PEG tube would be the best assignment for the LPN/VN. This is a nursing action that can be performed by the LPN/VN and does not require verification nor a co-signature by the RN. 1. Incorrect: The LPN/VN can collect data on a new client, but the RN must verify the information and co-sign the assessment. New admits require initial observation and data collection. From this, the RN must evaluate the information and formulate priorities of care. 2. Incorrect: Administering morphine IVP is out of the scope of practice for the LPN/VN. Therefore, this task should not be assigned to the LPN/VN. 4. Incorrect: Reinserting a PICC line is out of the realm of practice for an LPN/VN. Therefore, this task should not be assigned to the LPN/VN.

What important principle should the nurse reinforce with the client performing intermittent self-catheterization? You answered this question Correctly 1. Inserted in an emergency department. 2. Used to treat urinary catheter infections. 3. Is a clean procedure. 4. Requires use of sterile gloves.

3. Correct: Intermittent catheterization is a clean, not sterile, technique when performed in the home environment. Home intermittent catheterization is preferred over continuous use of a Foley catheter, as a Foley catheter increases client risk of urinary tract infections. 1. Incorrect: The client can be taught to do self-catheterization at home. An intermittent self-catheterization does not need to be performed in an emergency department. 2. Incorrect: Performing intermittent self-catheterization at home is recommended for urinary retention. This procedure does not treat urinary tract infections. 4. Incorrect: This procedure is a clean, not sterile, technique when performed in the home environment. Sterile gloves are not required.

The nurse is monitoring the healing of a full-thickness wound to a client's right thigh. The wound has a small amount of blood during the wet to dry dressing change. What action should the nurse initiate next? You answered this question Incorrectly 1. Notify the primary healthcare provider. 2. Obtain wound culture. 3. Document the findings. 4. Remove dressing and leave open to air.

3. Correct: Look at the clues: full thickness wound, small amount of blood, and wet to dry dressing. With a full thickness wound, there is destruction of the epidermis, dermis, and subcutaneous tissues going down to the bone. ​So you would expect to see a small amount of blood, or drainage wouldn't you? Yes. This is expected. Simply document this normal finding. 1. Incorrect: Is there really anything to worry about in this situation? No, so you do not need to notify the healthcare provider. Now, with most questions on NCLEX, there is something to worry abou,t but just not with this one. 2. Incorrect: No, bleeding is not a sign of infection, which is what you would be worried about if you got a wound culture. 4. Incorrect: You probably would not remove the dressing and leave the wound open to air. The small amount of blood noted is just a sign of blood flow in the healing wound. Wet to dry dressings help to debride the wound. So, if you remove the dressing, will debridement occur? No.

The nurse is preparing a client for a renal biopsy. Which data is most important to gather prior to this procedure? You answered this question Incorrectly 1. BUN 2. NPO status 3. Prothrombin time (PT) 4. Serum potassium 5. Activated partial thromboplastin time (aPTT)

3.&5 Correct: Yes, before you insert a needle into an organ for a biopsy it would be best to know their bleeding time and coagulation studies. Prothrombin is a protein produced by the liver. The PT test measures how well and how long it takes for blood to clot. Normally this is 20-30 seconds. aPTT also measures the time it takes for blood to clot. 1. Incorrect: Although BUN is related to renal function, it does not impact the procedure itself. This is not essential for the procedure. 2. Incorrect: Although this is carried out, it does not take priority over the risk of bleeding. Always think what could be life threatening. 4. Incorrect: Although serum potassium is related to renal function, it does not impact the procedure itself. This is not essential for the procedure.

A client with a history of congestive heart failure has an implantable cardioverter defibrillator (ICD) surgically implanted. What teaching points should the nurse reinforce with the client prior to discharge? You answered this question Incorrectly 1. Avoid hot baths and showers. 2. Increase intake of leafy green vegetable products. 3. Avoid magnets directly over the site. 4. Notify primary healthcare provider whenever a shock is delivered by the ICD. 5. Driving is not recommended for 1 year after placement of an ICD.

3., & 4. Correct: Magnets can deactivate the defibrillator. Other transmitter devices should also be avoided. Most arrhythmias need only one shock, but the healthcare provider should be notified when a shock is delivered so that monitoring can increase. 1. Incorrect: Hot baths or showers are not contraindicated with ICDs. 2. Incorrect: Increase of leafy green vegetable products would have no relation to the ICD but should be avoided if the client is on warfarin. 5. Incorrect: The client cannot drive for 6 months after implantation of an ICD and cannot drive for 6 months after any shock therapy from the ICD.

A licensed practical nurse (LPN) is utilizing the nursing process to care for assigned clients. Which nursing actions should the LPN relate to the implementation step of the nursing process? You answered this question Incorrectly 1. Collecting client data for a nursing history. 2. Reporting client response to a new medication. 3. Procuring equipment for a planned medical procedure. 4. Assigning client care activities to unlicensed assistive personnel. 5. Delivering skilled nursing care according to an established health care plan.

3., 4., & 5. Correct: The nurse should relate procuring medical equipment, assigning client care activities, and delivering skilled nursing care to the implementation step of the nursing process. Implementation is the third step of the nursing process and consists of delivering nursing care according to an established health care plan and as assigned by the RN or other person(s) authorized by law. 1. Incorrect: This is not the implementation step of the nursing process. LPNs participate in the assessment step of the nursing process by collecting client data for a nursing, psychological, spiritual, and social histories, comparing the data collected to normal values and findings. 2. Incorrect: This is not the implementation step of the nursing process. LPNs participate in the nursing process by reporting client responses to the RN or supervising healthcare provider.

The nurse is providing care to a client who has returned to the long-term facility following cataract surgery. Which finding would indicate a possible complication? You answered this question Correctly 1. Minimally swollen eyelid 2. Mild discomfort of the eye 3. Slight red appearance of the eye 4. Extreme pain in the eye

4. Correct. The postoperative cataract client usually experiences little to no pain, and it can be managed with mild analgesics. If the pain is severe, there may be an increase in intraocular pressure, hemorrhage, or infection, and the surgeon should be notified. 1. Incorrect. Slight swelling of the eyelid is considered a normal finding following cataract surgery. 2. Incorrect. The postoperative cataract client usually experiences little to no discomfort following surgery. This is a normal finding. 3. Incorrect. Slight redness is an expected finding. Pay attention to the word "slight". Increased redness is cause for concern. Compare it to the non-operative eye.

The nurse makes selections from the hospital menu for a client who is confused and suspicious of others. Which menu choice is best? You answered this question Correctly 1. Ham and vegetable casserole 2. Cheese and crackers 3. Caffeine free tea 4. Packaged sugar free Jell-O

4. Correct: A client who is suspicious of others needs foods that are packaged and can see them opened. 1. Incorrect: A client who is suspicious of others needs to be able to identify the ingredients in the food that is being eaten. A casserole contains many ingredients, and the client may fear that something has been added to the food. 2. Incorrect: Finger foods are best for clients that are manic. 3. Incorrect: Drinks and foods with no caffeine are okay for the confused and suspicious client, but this menu choice is not the best choice from the list here.

A client diagnosed with alcoholism was admitted to the medical unit with substance-withdrawal delirium. Two days later, the client decides to leave the hospital against medical advice. What action should the nurse take? You answered this question Correctly 1. Hide the client's clothes so that the client cannot leave. 2. Administer the ordered sedative. 3. Place restraints on the client. 4. Determine why the client wants to leave.

4. Correct: Always assess why the client wishes to leave first. This is the only way to fix the problem. 1. Incorrect: By confining a client against his or her wishes, and outside of an emergency situation, the nurse may be charged with false imprisonment. Actions that may invoke these charges include locking an individual in a room, taking a person's clothes for the purposes of detainment, and restraining in mechanical restraints a competent, voluntary client who demands to be released. 2. Incorrect: By confining a client against his or her wishes, and outside of an emergency situation, the nurse may be charged with false imprisonment. Actions that may invoke these charges include locking an individual in a room, taking a person's clothes for the purposes of detainment, and restraining in mechanical restraints a competent, voluntary client who demands to be released. 3. Incorrect: By confining a client against his or her wishes, and outside of an emergency situation, the nurse may be charged with false imprisonment. Actions that may invoke these charges include locking an individual in a room, taking a person's clothes for the purposes of detainment, and restraining in mechanical restraints a competent, voluntary client who demands to be released.

The nurse is caring for a client with hyperemesis gravidarum. What electrolyte imbalance is most likely? You answered this question Correctly 1. Hypocalcemia 2. Hypomagnesemia 3. Hyponatremia 4. Hypokalemia

4. Correct: Hyperemesis gravidarum is characterized by persistent, severe pregnancy related nausea and vomiting. There is a large amount of potassium in the upper GI tract. A client with prolonged vomiting will lose potassium in the emesis. Additionally, the client is unable to replace the lost potassium due to the persistent nausea and vomiting. 1. Incorrect: Hypocalcemia results from any condition that causes a decrease in the production of parathyroid hormone (PTH). Hyperemesis gravidarum does not affect PTH levels. 2. Incorrect: The lower GI tract has a lot of magnesium. Therefore, this client is at risk for hypomagnesemia, but not more than hypokalemia. The client with hyperemesis gravidarum is losing upper GI contents. 3. Incorrect: The client with hyperemesis gravidarum is at high risk for being dehydrated. The electrolyte imbalance associated with dehydration is hypernatremia, not hyponatremia. Remember, this client's blood will be concentrated, and concentrated makes numbers go up (i.e. sodium, hematocrit, and specific gravity).

The client needs assistance to apply anti-embolism stockings each day in the long-term care facility. Today, as the nurse enters the room to apply the stockings, she finds that the client has been walking about the unit for 30 minutes. What should the nurse do first to lessen the risk of swelling of the lower extremities? You answered this question Correctly 1. Ask the client to lie down and place the stockings on the legs. 2. Ask the client to sit on the bedside and place the stockings on the legs. 3. Tell the client that the nurse will return later to assist with the application. 4. Elevate the extremities in bed for 30 minutes before application.

4. Correct: The client should have extremities elevated to encourage venous return and reduce the risk of swelling before the stockings are applied. 1. Incorrect: To place the stockings on immediately will cause further venous stasis and swelling. 2. Incorrect: The extremities should be elevated for a period of time before application. 3. Incorrect: This instruction alone does not give the client adequate information about the need to keep the lower extremities elevated before applying the stockings.

The nurse is preparing the sterile field to assist the primary healthcare provider with a procedure. Which flap of the sterile pack should the nurse open first? You answered this question Incorrectly 1. Closest to the nurse. 2. To the left of the nurse. 3. To the right of the nurse. 4. Farthest from the nurse.

4. Correct: The flap farthest from the nurse should be opened first so that the nurse's arm or hand does not cross the sterile field. 1. Incorrect: The flap closest to the nurse should be opened last so that the sterile field is not crossed by the nurse's arm or hand. 2. Incorrect: The sides should be opened in the 2nd and 3rd steps so that the nurse's hand does not cross the sterile field. 3. Incorrect: The sides should be opened in the 2nd and 3rd steps so that the nurse's hand does not cross the sterile field.

The client states, "I really do not want to have surgery. I have told my children this, but they still want me to go through with the surgery. I do not know what to do." What is the best response for the nurse as client advocate? You answered this question Incorrectly 1. "Your children are concerned about you. The surgery is the best thing for your health." 2. "You have some genuine concerns about the surgery, and you feel as if your children are not addressing your concerns. You and your family will need to resolve this before you go to surgery." 3. "I can contact your primary healthcare provider so that you can discuss your concerns regarding surgery." 4. "You have some genuine concerns about the surgery, and you feel as if your children are not addressing your concerns. Tell me more about your concerns."

4. Correct: The nurse has a duty to advocate for the client if there is a discrepancy between the care or proposed care and the client's wishes regarding treatment. It is important to acknowledge the client's feelings and to demonstrate compassion and a willingness to understand. This presents an opportunity for additional communication to help answer some of the client's questions or set up a client-family conference with the client, the client's family, and the primary healthcare provider. 1. Incorrect: When the nurse agrees with the client's children, the nurse ignores the client's feelings and does not address the issue of the client's treatment wishes. 2. Incorrect: When the nurse restates the client's comment without investigating the client's concerns, the issue goes unresolved. 3. Incorrect: Offering only to contact the primary healthcare provider is an incomplete solution and hints of the nurse not taking responsibility to investigate the client's concerns. The client may be uncomfortable addressing concerns with the primary healthcare provider before resolving the issue of treatment wishes with family members.

The nurse is caring for a client 28 weeks pregnant who reports swollen hands and feet. Which additional sign or symptom would cause the greatest concern? You answered this question Correctly 1. Nasal congestion 2. Hiccups 3. Blood glucose of 130 4. Muscle spasms

4. Correct: This client could have preeclampsia and would be at risk for seizures. 1. Incorrect: This is a common occurrence during pregnancy and is not the greatest concern. 2. Incorrect: Hiccups would be the second best answer, indicating nerve/muscle irritation, but not a common symptom associated with preeclampsia. 3. Incorrect: Not the greatest concern with presenting signs and symptoms of swelling. The blood glucose is elevated, but the priority data is the possibility of muscle spasms which may progress to seizures due to eclampsia.

The nurse, caring for an 8 month old infant, should recognize which major stressor of hospitalization for this infant? You answered this question Correctly 1. Fear of unknown 2. Loss of daily routine 3. Body image disturbance 4. Separation anxiety

4. Correct: Yes, they are afraid of being without the caregiver. Separation anxiety develops after a child gains an understanding of object permanence. The infant may become unsettled after the parent leaves. Although some babies display object permanence and separation anxiety as early as 4 to 5 months of age, most develop more robust separation anxiety at around 8 months. Separation anxiety can be worse if the infant is hungry, tired, or not feeling well. 1. Incorrect: Fear of the unknown is not a concern at this age, but rather between the age of 2-3 years. Separation anxiety is their immediate concern. 2. Incorrect: Keeping family routines and providing quality time with trusted adults is reassuring once the child reaches the age of 2. 3. Incorrect: The preschooler fears mutilation resulting in body image disturbance.

A client who has been on bed rest for several days is ambulating for the first time with assistance. Prioritize the actions the nurse should take by placing them in order from first to last.

In order to keep a client safe, the nurse should first check the client's orientation to determine the client's ability to follow instructions. Second, to avoid orthostatic hypotension, the nurse should assist the client to sit on the side of the bed. Third, apply the gait belt to ensure safety while ambulating. Fourth, assist the client to stand for a few seconds. The fifth action is to ambulate in the room.

A home care nurse is visiting a client who delivered her first baby one week ago. What behavior by the client would indicate to the nurse that maternal-infant bonding is occurring? You answered this question Correctly 1. Holds baby face to face 2. Talks about the baby's features 3. Touches baby frequently 4. Talks to baby 5. Allows baby to cry vigorously for 15 minutes

Options 1, 2, 3, and 4 are all true. A positive maternal-infant bond has formed when baby and mom become intimately involved with each other through behaviors and stimuli that are complementary and provoke further interactions. These options illustrate such behaviors. Option 5 is false. This is not a behavior that is seen when there is a positive maternal-infant bond.

The nurse is caring for a client who has a history of sleep apnea. The client is scheduled for a colon resection the following morning and asks if the sleep apnea machine should be brought to the hospital. What is the nurse's best response? You answered this question Correctly 1. Yes, bring the sleep apnea machine. 2. No, do not bring the sleep apnea machine. 3. It is your choice. 4. Call your primary healthcare provider.

Sleep apnea is a concern for clients who are about to have surgery. These clients are at an increased risk for developing respiratory and cardiovascular complications in the postoperative period. Complications can include irregular heart rhythms, oxygen deficiency, high blood pressure, diabetes, stroke, heart attack, and even death. Positive airway pressure (PAP) is considered the gold standard treatment for sleep apnea. With continuous positive airway pressure (CPAP), the client will wear a mask over the nose and/or mouth while sleeping. An air blower will force air through the airway, which will prevent it from closing during sleep. The nurse should closely monitor the client's vital signs both during and after surgery

The LPN/VN is preparing to transfer a client from the delivery room to the postpartum unit. Which statement by the client would be the priority for the LPN/VN to notify the charge nurse? You answered this question Correctly 1. "I just felt something gush." 2. "I feel like I am still having contractions." 3. "When I stand up, I feel dizzy for several moments" 4. "My hemorrhoids are hurting."

1. Correct. This could indicate postpartum hemorrhage (PPH) and requires immediate assessment by the nurse. PPH can be caused by the following: placenta previa, cervical lacerations, vaginal tear, or a ruptured or inverted uterus. 2. Incorrect. This is normal postpartum contractions of the uterus to help dispel clots and to return the uterus to normal size. The contractions may occur for several days after delivery. 3. Incorrect. Due to the fluid loss during the delivery, the client may be experiencing orthostatic hypotension. Teach safety measures. Although the LPN would need to inform the charge nurse of this, it is not the priority over the report that may indicate postpartum hemorrhage. 4. Incorrect. The client's hemorrhoids can be painful after delivery. However, this is not the priority to report. Remember, bleeding would be a priority over pain.

Following surgery, a client is placed on methylprednisolone. What additional drug therapy would the nurse expect to be prescribed for this client to prevent an adverse reaction related to methylprednisolone? You answered this question Correctly 1. Pantoprazole 2. Phenytoin 3. Imipramine HCI 4. Aminocaproic acid

1. Correct: A potential side effect of methylprednisolone is a peptic ulcer. The primary healthcare provider will prescribe a proton pump inhibitor or H2 blocker to prevent this side effect. 2. Incorrect: Phenytoin is an anticonvulsant. Seizures are not a side effect of methylprednisolone. 3. Incorrect: Imipramine HCI is an antidepressant which is not routinely given with methylprednisolone (Although mood changes can occur with steroid administration, anti-depressants are not routinely given). 4. Incorrect: Aminocaproic acid is given when clients are bleeding. Bleeding is not a side effect of methylprednisolone

Which action, if done by a new LPN/VN, needs to be interrupted by the precepting LPN/VN? You answered this question Correctly 1. Mixes diazepam and hydromorphone in one syringe. 2. Administers diazepam before meals. 3. Raises side rails after administering hydromorphone. 4. Instructs client to call for assistance getting out of bed after administration of diazepam.

1. Correct: In this question, you are looking for the answer that is unsafe and should not be done. Diazepam cannot be mixed with any other medication. The charge nurse should intervene. 2. Incorrect: This is an appropriate action. Food in the stomach delays absorption of diazepam, so it would need to be given before meals. 3. Incorrect: This would be an appropriate action. Hydromorphone is a narcotic and can decrease level of consciousness (LOC) and increase the risk of falls, so the nurse would be taking appropriate measures to ensure the client's safety. 4. Incorrect: This would be an appropriate action. Diazepam relaxes the muscles, decreases LOC, and can increase the risk of falls.

A young client experiencing a manic episode tells the night nurse, "If you do not go to bed with me, I am going to have you fired." Which statement by the nurse is appropriate? You answered this question Correctly 1. "That is inappropriate behavior and you will need to go to your room." 2. "You've got to be kidding! You can't get me fired." 3. "I don't want to hear that again; don't ever say that again." 4. "I can see that you need attention, but this is not the way to get it."

1. Correct: Set limits on manipulative behaviors. Explain what is expected and what the consequences are if limits are violated. 2. Incorrect: Do not argue with the client. 3. Incorrect: This is confrontational and does not set consequences. 4. Incorrect: Remember to set limits and do not encourage this behavior.

Which nursing task would be appropriate for the LPN/VN to complete? You answered this question Correctly 1. Obtain a wound culture from a client. 2. Administer regular insulin IV to a client in diabetic ketoacidosis. 3. Update plan of care for a client. 4. Initiate client teaching on ostomy

1. Correct: The LPN/VN has the knowledge and skill to obtain a wound culture. This is within the scope of practice for the LPN/VN. 2. Incorrect: The LPN/VN cannot administer IV medications to an unstable client. This client needs the RN for close monitoring. 3. Incorrect: The LPN/VN cannot develop or update a plan of care for a client. This is an RN task. The PN can contribute to the development and update, however. 4. Incorrect: Initiating teaching is the task of an RN. LPN/VN's can collect data, observe, monitor the client, and reinforce teaching

Which primary healthcare prescription should the nurse perform first? You answered this question Correctly 1. Insert intermittent catheter in client who has not voided in 8 hours. 2. Administer a bolus feeding via a client's gastrostomy tube. 3. Reinsert nasogastric tube (NG) that was pulled out. 4. Remove wound sutures.

1. Correct: The client who has not voided after 8 hours needs to be catheterized. This is the priority since the bladder is likely full and could lead to backflow of urine to the kidneys. 2. Incorrect: Not the priority here. The feeding can be done after the catheter is inserted into the client who cannot void. 3. Incorrect: Again, not the priority. This can be done after the catheter is inserted. 4. Incorrect: Removing sutures is not a priority. This could be done last.

The nurse is preparing to administer iron dextran IM. Which injection site would be best for administration? You answered this question Correctly 1. Ventrogluteal 2. Vastus lateralis 3. Rectus Femoris 4. Deltoid

1. Correct: This site would be used for Z track IM injections. Iron preparations are administered by the Z track technique, preferably in a large, deep muscle such as the ventrogluteal muscle. 2. Incorrect: The vastus lateralis site could be used in adults with enough muscle mass, but the ventrogluteal is the preferred site. 3. Incorrect: The rectus femoris site can be used in adults when other sites are no longer accessible. It is not the preferred site. 4. Incorrect: The deltoid is a small muscle that is not well developed in many clients. It is not a recommended site for Z track medication administration.

How should the nurse prepare a client for a paracentesis? You answered this question Incorrectly 1. Place client in the Fowler's position. 2. Position client flat with right arm behind the head. 3. Ask the client to empty bladder. 4. Obtain client's vital signs every 4 hours. 5. Maintain NPO status for 4 hours pre-procedure.

1., & 3. Correct: The correct position is HOB elevated to allow fluid to pool in one spot for the paracentesis. The nurse knows this is a lower abdominal puncture and the bladder should be empty to avoid puncturing the bladder. 2. Incorrect: The optimal position is HOB elevated to allow the fluid to pool in one spot. If the nurse were to lie the client flat, the fluid would go everywhere. 4. Incorrect: Obtain a set of vital signs immediately prior to the procedure and immediately after the procedure. Vital signs every 4 hours will not give you needed data on the client's status. 5. Incorrect: This procedure does not require NPO status. NPO status is initiated when there is a risk of aspiration during or following the procedure.

A client, who is having difficulty falling asleep, asks the nurse for a sleeping aid. What is the first action the nurse should provide to the client? You answered this question Correctly 1. Assist client to take a cool bath. 2. Provide a back massage. 3. Administer prescribed triazolam. 4. Give client a crossword puzzle to work.

2. Correct: Research indicates that back massage can enhance client comfort, relaxation, and sleep. This is the least invasive option and should be done first. 1. Incorrect: A cool bath would wake a client, whereas, a warm bath would increase relaxation. 3. Incorrect: Triazolam is a short acting benzodiazepine. Do not go to the sleeping pill first. 4. Incorrect: Distraction is a good strategy for drawing a client's attention away from pain but may increase thinking, thus keeping the client awake.

What discharge instructions should the nurse reinforce to the client post abdominal hysterectomy? You answered this question Incorrectly 1. Ambulate at least 3-4 times per day. 2. Notify the primary healthcare provider of a yellow discharge from the surgical wound. 3. Swimming is allowed if staples were used to close the skin. 4. Press a pillow over incision when coughing to ease discomfort. 5. Apply moist heat to surgical site the first couple of days for pain relief.

1., 2., & 4. Correct: The client should get up and move to prevent complications such as deep vein thrombosis (DVT), pneumonia, constipation, etc. The healthcare provider should be notified if the surgical wound is bleeding, red and warm to touch, or has a thick, yellow, or green drainage. Pressing a pillow over incision when coughing or sneezing will ease discomfort and protect the incision. 3. Incorrect: Do not go swimming or soak in a bathtub or hot tub until the primary healthcare provider says it is ok. You worry about infection. 5. Incorrect: In the first couple of days, an ice pack may help relieve some pain at the site of surgery. Remember, NCLEX wants you to think safety first when it comes to the use of heat.

What should the nurse document after a client has died? You answered this question Incorrectly 1. Time of death 2. Who pronounced the death 3. Disposition of personal articles 4. Destination of body 5. Primary healthcare provider's prescriptions 6. Time body left facility

1., 2., 3., 4., & 6. Correct: All of these are correct options that should be documented. In addition to these things, the nurse should also document consideration of and preparation for organ donation, family notified and decisions made, and location of identification tags. 5. Incorrect: The primary healthcare provider's prescriptions do not need to be documented after a client dies.

A client who is obese and paraplegic needs to be repositioned in the bed. What actions should the nurse take? You answered this question Correctly 1. Obtain assistance from a coworker. 2. Place the bed in the lowest position with the client close. 3. Adjust the bed to a workable position and move close to the client. 4. Use a draw sheet with the assistance of a coworker and pivot the hips while pulling the draw sheet upward. 5. Use the client's arms and pull to head of bed to aid positioning.

1., 3. & 4. Correct: The nurse should solicit a coworker for help, adjust the bed to a workable position, move close to the client, use a draw sheet with the assistance of a coworker, and pivot the hips while pulling the draw sheet upward. These steps will prevent injury to the nurse and client. 2. Incorrect: The bed needs to be adjusted to the nurse's working height, not in the lowest position. 5. Incorrect: The client's arms should not be pulled on or used to position a client.

Following a total hip replacement, the nurse reinforces discharge teaching to the client. The nurse knows that reinforcement of teaching was effective when the client states which activities are safe to perform? You answered this question Correctly 1. Using an abduction pillow while sleeping 2. Crossing the legs 3. Using a toilet extender 4. Showering rather than taking a bath 5. Tying shoes

1., 3., & 4. Correct: The client should use an abduction pillow to keep hip in proper alignment and prevent hip dislocation. A toilet extender keeps the hip in proper alignment and prevents hip dislocation. Showering, rather than sitting in a tub, will prevent flexion of the hip. 2. Incorrect: Crossing the leg can pop the hip out of place and prevent total healing and success with the replacement. 5. Incorrect: To tie shoes, the client has to bend over which can pop the hip out of place. The client would need to have shoes that do not require tying or have someone do it for them.

A client with schizophrenia tells the nurse, "I want you to take me to the uniphorum". Which statement would be most appropriate for the nurse to make? You answered this question Correctly 1. "You don't even know what you are saying. Stop making up words". 2. "I don't understand what you mean by that. Would you please explain it to me"? 3. "Think about what you are trying to say, then try again". 4. "I will take you after I finish handing out medications".

2. Correct: Attempt to decode incomprehensible communication patterns. Seek validation and clarification. These techniques reveal to the client how he or she is being perceived by others, and the responsibility for not understanding is accepted by the nurse. 1. Incorrect: Do not argue or belittle clients. This is nontherapeutic. This places all responsibility for communication on the client and suggests that the nurse believes the client is defective. This would likely frustrate and distress the client and reduce self-esteem. 3. Incorrect: Cognitive impairment is persistent rather than momentary, so asking the client to do this will be difficult. 4. Incorrect: Pretending to understand is nontherapeutic because it gives the false impression that the client is communicating effectively.

Which menu selection by the client diagnosed with nephrotic syndrome indicates that reinforcement of dietary teaching was understood? You answered this question Correctly 1. Pancakes with whipped butter, syrup, bacon, apple juice 2. Scrambled eggs, sliced turkey, biscuit, whole milk 3. Grits, fresh fruit, toast, coffee 4. Bagel with jelly, hash browns, tea

2. Correct: Client needs low sodium and increased proteins. 1. Incorrect: This selection is too high in sodium and fats. 3. Incorrect: This selection has no protein. Remember, nephrotic syndrome is the exception to the rule of limiting protein. These clients need increased protein to compensate for the large loss of protein in the urine. 4. Incorrect: This selection has no protein. Remember, nephrotic syndrome is the exception to the rule of limiting protein. These clients need increased protein to compensate for the large loss of protein in the urine.

What precautions should be taken with computer monitors that display client health information to ensure client's confidentiality? You answered this question Incorrectly 1. Turn the screen facing the client rooms so that healthcare personnel can access the information easily. 2. Have the screen placed facing away from any visitor or client care area. 3. Turn the computer monitors off when the computer is not in use. 4. The computer should be kept in a secured, locked area.

2. Correct: Computer monitors that display client health information should be positioned away from the view of any visitors or unauthorized persons. Even a well-guarded computer monitor, with an authorized employee sitting in front of it, could be a potential breach of confidentiality, depending on the angle of the monitor screen and who was attempting to view the information on it. The responsibility for keeping health information safe is on every member of the healthcare team. 1. Incorrect: No, this would be easily viewed by unauthorized individuals. 3. Incorrect: Not necessary to turn off if proper precautions are taken. 4. Incorrect: Not necessary to keep in secured area if proper precautions are taken.

Which electrolyte imbalance would be the nurse's priority concern in the burn client? You answered this question Correctly 1. Hypernatremia 2. Hyperkalemia 3. Hypoalbuminemia 4. Hypermagnesemia

2. Correct: Good job. When the cells lyse, they release potassium, and then the serum potassium goes up. And if the kidneys stop functioning, we are in real trouble. 1. Incorrect: Well, hypernatremia does occur when the client becomes very dehydrated, but it's not as dangerous as the potassium one. 3. Incorrect: Low albumin can cause problems keeping fluid in the vascular space, but albumin is not an electrolyte. 4. Incorrect: No, the magnesium doesn't go up unless the kidneys shut down.

A client being treated for osteoporosis with alendronate reports experiencing slight heartburn after taking the medicine. What information should the nurse reinforce to help reduce this side effect? You answered this question Correctly 1. Stop taking the medication. 2. Drink plenty of water with the medication. 3. Take the medication before bedtime. 4. An antacid should be taken with the medication.

2. Correct: Increased heartburn can be reduced or prevented by drinking plenty of water, sitting upright following the administration of the medication, and avoiding sucking on the tablet. 1. Incorrect: There is no need to stop the medication due to mild heartburn. Measures such as those listed above should be utilized to minimize heartburn. However, if extreme pain or difficulty swallowing develops, this should be reported to the primary healthcare provider. 3. Incorrect: The client should take the medication in the morning, thus preventing prolonged contact with the esophagus. 4. Incorrect: The absorption of the medication is decreased when it is taken with calcium, iron, and magnesium, or antacids containing calcium, aluminum, or magnesium. Thirty minutes should elapse before taking the antacid following administration of the alendronate.

Which action by a new nurse indicates to the supervising nurse that the sterile field has been contaminated? You answered this question Correctly 1. Maintains the sterile field above the level of the waist. 2. Places sterile gauze dressing within the one inch border of sterile field. 3. Remains facing the sterile field throughout procedure. 4. Inspects sterile wrapped instruments for damage.

2. Correct: No sterile object should be within the one inch border of the sterile field, as the object is no longer considered sterile. 1. Incorrect: This is a correct action. Bacteria tend to settle below the level of the waist, so there is less contamination when the field is above the waist and away from the nurse. 3. Incorrect: This is a correct action. The nurse should never turn their back to the sterile field. The fronts of sterile gowns are considered sterile from the chest to the level of the sterile field. 4. Incorrect: This is a correct action. The sterile wrapped instruments and trays should be purposely inspected for small tears that would compromise sterility before opening and placing the instruments on the field

The client in the manic phase of bipolar disorder begins climbing onto a table in the dayroom and shouts, "I can fly! I can fly! Watch me fly!" What should be the initial intervention by the nurse? You answered this question Correctly 1. Leave the client alone and remove clients from the dayroom. 2. Call for personnel to escort the client out of the day room. 3. Restrain the client, and notify the primary healthcare provider. 4. Tell the client that there is no way that a person can fly.

2. Correct: The first priority is the safety of the client. If the client jumps off the table, this action may cause an injury to the client. The nurse may need extra help in case the client becomes violent. 1. Incorrect: The client may cause self injury by jumping from a table and should not be left alone. The other clients can be moved away from the table or removed from the dayroom. 3. Incorrect: The primary healthcare provider does not need to be notified at this time. Restraints should be used as a last resort. 4. Incorrect: The safety of the client at this time is the priority intervention. Presenting a client with reality is a therapeutic communication technique.

The nurse has observed that the client on the skilled nursing unit has been consuming fewer calories over the past three days. There has been no other change in the client's condition. Which intervention is most important for the nurse to initiate? You answered this question Correctly 1. Suggest that the family seek an appointment with the primary healthcare provider. 2. Ask the dietician to visit the client and discuss food preferences. 3. Note any weight loss over the next month. 4. Continue to monitor intake over the next couple of weeks.

2. Correct: The nurse is using the expertise of other team members by requesting that the dietician visit the client. This is the most important measure to address the client's nutritional needs. The problem may be that the client simply does not like the foods that have been served, and the dietician is the best one to address these issues. 1. Incorrect: An appointment with the primary healthcare provider may not be necessary. It is best to first utilize available team members such as the dietician. The nurse would then notify the primary healthcare provider of any pertinent findings. 3. Incorrect: To simply monitor weight loss for a month would not be an appropriate intervention. There could be significant weight loss within a month. This is much too long to wait before taking measures to ascertain the reason for the client consuming fewer calories.

A nurse drops a glass bottle, which shatters on the floor in the hallway. What action should the nurse take? You answered this question Correctly 1. Notify housekeeping to clean up the spill. 2. Pick up glass with a broom and dustpan and dispose into a puncture resistant sharps container. 3. Pick up the glass with gloved hands and dispose into a puncture resistant sharps container. 4. Use a wet mop to collect the glass and dispose of it in the garbage can.

2. Correct: The nurse must not be cut by the broken glass. Proper removal of glass includes using a dustpan and broom to collect the glass and disposing of it into a puncture resistant sharps container. 1. Incorrect: While waiting for housekeeping, someone could fall or get cut. The nurse should initiate cleanup. 3. Incorrect: Hands are never used to pick up glass, even if they are gloved, because of the increased risk of getting cut. 4. Incorrect: A wet mop will not pick up the glass, and glass pieces will stick to a wet mop.

Which action by two unlicensed nursing personnel (UAPs), while moving the client back up in bed, would require intervention by the nurse? You answered this question Correctly 1. Lowers the side rails closest to them. 2. Places hands under client's axilla. 3. Lowers the head of bed. 4. Raises the height of the bed.

2. Correct: This action is not appropriate and requires intervention by the nurse. This could damage the brachial plexus nerves under the axilla. Use a draw sheet to prevent this from occurring. 1. Incorrect: This is a correct action. The UAPs will need to lower the side rails closest to them to safely move the client up in bed. Not lowering the rails could result in injury to the UAPs' backs. 3. Incorrect: This action is correct. Moving the client upward with the head of the bed raised works against gravity, requires more force, and can cause back strain. 4. Incorrect: This action is appropriate and would not require intervention by the nurse. Raising the height of the bed brings the client close to the UAPs center of gravity and decreases the chance of back injury

A nurse is caring for a Mexican-American client post stroke. While in the client's room, a curandero visits at the request of client. What is the best action of the nurse? You answered this question Incorrectly 1. Leave, and return once the curandero has left. 2. Reinforce client care with the client and curandero. 3. Ask the curandero to leave so that the client can be observed. 4. Explain to the client that the curandero is not a reliable healthcare option.

2. Correct: This is the best course of action for the nurse. The health and healing of a client come from many components, including spirituality, religion, folk remedies, alternative therapies, and modern medicine. Unless something is harmful to the client, it is best to incorporate all components into the care of the person. 1. Incorrect: Leaving will not allow the nurse to discuss and reinforce care of the client with all members of the healthcare team and family. This is a good time to learn about the curandero, health beliefs, etc. 3. Incorrect: The client and family have requested the curandero. Asking him to leave would be insulting. The nurse would not develop a good rapport with the client this way. 4. Incorrect: This does not take into account the client's beliefs in health, wellness, and illness. The nurse should work to incorporate folk medicine from the curandero as long as it will not harm the client.

Which tasks would be appropriate for the LPN/LVN to assign to an unlicensed assistive personnel (UAP)? You answered this question Incorrectly 1. Ask the client diagnosed with dementia memory-testing questions. 2. Collect the urinary output hourly on the client with renal disease. 3. Demonstrate pursed lipped breathing to the client who has emphysema. 4. Give a tepid sponge bath to the client who has a fever. 5. Assess oxygen saturation on a client experiencing angina.

2., & 4. Correct: The UAP can obtain hourly urine output on clients and can give a tepid sponge bath to a client. The LPN/VN must know what tasks can be assigned to the UAP. 1. Incorrect: The nurse cannot delegate assessment, evaluation, or teaching. This would be an assessment function for the RN to perform. 3. Incorrect: The UAP cannot teach. Demonstration is a method of teaching. This is an RN task. 5. Incorrect: The UAP cannot assess the client experiencing angina. This is an RN task.

The nurse is caring for a client with decreased cardiac output secondary to heart failure with fluid volume overload. Which signs/symptoms are an indication to the nurse that treatment has not been effective? You answered this question Incorrectly 1. Diuresis 2. Dyspnea on exertion 3. Persistent cough 4. Warm, dry skin 5. Heart rate irregular at 118/min 6. Alert and oriented

2., 3. & 5. Correct: When the cardiac output decreases, renal perfusion decreases, which leads to decreased urine output and fluid retention. This leads to difficulty breathing. Tachycardiac and rhythm irregularity are signs of fluid volume excess (FVE) and decreased output. Persistent cough, wheezing, and pink blood tinged sputum are all signs that the client is still sick. 1. Incorrect: Diuresis is what we want, which indicates that the treatment for FVE is effective. 4. Incorrect: Indication of improved cardiac output. 6. Incorrect: Indication of improved cardiac output.

A client is diagnosed with seizures. Which nursing interventions should the nurse implement? You answered this question Incorrectly 1. Have an unlicensed assistive personnel stay with the client. 2. Pad the side rails with blankets. 3. Place the bed in low position. 4. Keep a padded tongue blade at the bedside. 5. Instruct client to call for help to ambulate.

2., 3., & 5. Correct: Place a call light within reach, put the client close to the nurses' station, and pad the side rails. Have client call for assistance to BR, maintain bedrest until seizures are controlled, or ambulate with assistance to protect from injury. These interventions will help to protect the client from injury. 1. Incorrect: It is not necessary to have someone stay with this client at all times. After implementing the safety issues and transferring the client closer to the nurses' station, the client can stay alone. 4. Incorrect: Do not stick anything in a client's mouth during a seizure. A padded tongue blade could cause injury

A LPN/VN is caring for a client who reports a pain level of 8 on a numeric scale of 1-10. The LPN/VN reports the client's pain level to the RN and administers pain medication as prescribed. Which actions should a nurse take to advocate for this client? You answered this question Correctly 1. Notify the primary healthcare provider. 2. Ensure that bed side rails are raised and locked. 3. Administer naloxone within 30 minutes. 4. Advise the client to call for assistance before getting out of bed. 5. Monitor the client's pain level after administering medication.

2., 4. & 5. Correct: To advocate for this client, the LPN/VN should ensure that client's bed side rails are up and in a locked position and should advise the client to call for assistance before getting out of bed, because pain medication increases the client's risk for falls. The LPN/VN should also monitor the client's pain level after an appropriate amount of time to ensure that the client's pain is under control. 1. Incorrect: The nurse does not need to notify the primary healthcare provider because the LPN/VN reported the client's pain to the RN and had a prescription for pain medication to administer to the client. 3. Incorrect: Naloxone is an opioid antagonist. The question does not identify any adverse symptoms following the administration of the pain medication.

What symptoms would the nurse anticipate in a client with a calcium level of 3.2 mg/dL (0.80 mmol/L)? You answered this question Incorrectly 1. Slowed deep tendon reflexes 2. Muscle rigidity and cramping 3. Hypoactive bowel sounds 4. Positive Chvostek's sign 5. Seizures 6. Laryngospasms

2., 4., 5., & 6. Correct: Normal serum calcium is 8.7 - 10.3 mg/dL (2.18 - 2.58 mmol/L). The client with a calcium level of 3.2 mg/dL (0.80 mmol/L) is hypocalcemic. With hypocalcemia, the muscle tone is rigid and tight. Therefore, the client may report muscle cramping. A hallmark sign of hypocalcemia is a positive Chvostek's sign, which is a twitching of facial muscles following tapping in the area of the cheekbone that is indicative of hyperirritability. The client may be at risk of having seizures due to the neuromuscular irritability. Prolonged contraction of the respiratory and laryngeal muscles causes laryngospasm and stridor and may result in cyanosis. 1. Incorrect: With hypocalcemia, the deep tendon reflexes are increased, not decreased. 3. Incorrect: The client with hypocalcemia tends to have increased gastric motility, resulting in hyperactive (not hypoactive) bowel sounds. In addition, abdominal cramping and diarrhea are common with hypocalcemia.

A client is hospitalized for chronic renal failure. The nurse will need to notify the primary healthcare provider concerning which findings? You answered this question Incorrectly 1. Sodium 135 mEq/L (135 mmol/L) 2. Potassium 5.8 mEq/L (5.8 mmol/L) 3. BP 100/70 4. No weight loss 5. Calcium 8.0 mg/dL (2 mmol/L)

2., 5. Correct: Normal K 3.5-5.0 mEq/L (3.5-5.0 mmol/L); Normal serum Ca 9.0-10.5 mg/dL (2.25-2.62 mmol/L)​. The abnormal lab results need to be reported. Hyperkalemia is a serious electrolyte disorder associated with chronic renal failure due to decreased excretion of potassium from the kidneys. Progressive decrease in kidney function affects mineral and bone metabolism. The low calcium needs to be reported. 1. Incorrect: Normal sodium 135-145 mEq/L (135-145 mmol/L). This is a normal finding and does not need to be reported. 3. Incorrect: Hypertension is a potential complication of chronic renal failure. A BP of 100/70 is not an abnormal finding and does not need to be reported. 4. Incorrect: The desired outcome for clients with chronic renal failure is no rapid increases or decreases in weight. This finding does not need to be reported.

Which assignments would be most appropriate for the LPN/VN to accept from the RN? You answered this question Correctly 1. Six year old with new onset diabetes. 2. Ten year old with pneumonia admitted two days ago. 3. Three month old admitted with severe dehydration. 4. Four year old admitted for developmental studies. 5. Twelve year old with post op wound infection taking oral antibiotics.

2.,4. & 5. Correct: The best assignments for the LPN/VN would be the child with pneumonia admitted two days ago and the child admitted for developmental studies. The twelve year old with post op wound infection taking oral antibiotics is also stable. 1. Incorrect: The diabetic requires much teaching and supervision. This is an unstable client that should not be assigned to an LPN/VN. 3. Incorrect: The child with dehydration will require close intravenous fluid (IVF) monitoring, assessment and evaluation of condition. This client is unstable and should not be assigned to an LPN/VN

A busy LPN instructs an unlicensed assistive personnel (UAP) to obtain daily weights on a client. The LPN provides initial direction for the task, monitors that the task is successfully completed, reviews the results of the daily weight, and reports the results to the RN. Which action has the LPN taken? You answered this question Incorrectly 1. Failed to supervise the actions of the UAP. 2. Improperly assigned a client care task. 3. Appropriately assigned a client care task. 4. Supervised the performance of a client care task. 5. Functioned outside of the LPN scope of practice.

3. & 4. Correct: The LPN appropriately assigned the performance of a client care task. The LPN appropriately supervised the performance of a client care task. 1. Incorrect: The LPN appropriately supervised the actions of the UAP. 2. Incorrect: The LPN appropriately assigned the task to the UAP. 5. Incorrect: It is within the scope of practice for the LPN to assign daily weights to a UAP if the UAP has been properly trained to carry out the assigned task and is supervised.

The nurse recognizes that treatment has been successful in resolving fluid volume excess based on which findings? You answered this question Incorrectly 1. Continued lethargy 2. Heart rate 112/min 3. Decreasing shortness of breath 4. BP 114/78 5. Increased thirst

3. & 4. Correct: Urinary output should increase with decreasing shortness of breath as the FVE is corrected, and BP should be normal. 1. Incorrect: Level of consciousness (LOC) should improve with perfusion to the brain. 2. Incorrect: Heart rate should decrease if FVE is corrected. 5. Incorrect: Thirst is a sign of FVD, rather than FVE, so increased thirst would not be pertinent for FVE.

A nurse is caring for a client who had a total hip replacement 2 days ago. What observation would be a priority concern for the nurse? You answered this question Correctly 1. Small amount of red drainage on the surgical dressing. 2. Continues to report pain in hip when being repositioned. 3. Temperature of 101.8°F (38.7°C). 4. Slight swelling in the leg on the affected side.

3. Correct: A low grade fever is normal following hip surgery but a temperature of 101.8ºF (38.7ºC) two days postoperatively is higher than the expected slight increase and should be a priority concern. The development of an infection is one of the major complications for clients following hip surgery. Therefore, fever that persists above 101ºF that is accompanied by chills, diaphoresis, or increasing drainage and odor from the incision should be reported. 1. Incorrect: A small amount of red, bloody drainage on the dressing is expected as part of the normal healing process. 2. Incorrect: Some pain during repositioning after hip surgery is normal and can generally be managed with analgesics. 4. Incorrect: Swelling in the operative leg is a normal part of the postoperative process after hip surgery. Normal swelling is lessened in the morning but tends to re-accumulate throughout the day. This can be minimized by elevating the client's legs or having the client lie down for approximately 45 minutes during the day.

What should the nurse do first when caring for a client who is being admitted with a diagnosis of meningococcal meningitis? You answered this question Correctly 1. Perform neurological checks. 2. Collect data for health history. 3. Institute droplet precautions. 4. Orient client to the room and procedures

3. Correct: Although all the options are appropriate, you should first place the client on droplet precaution to prevent the spread of meningococcal meningitis. 1. Incorrect: Placing client in isolation should be done first (actually prior to arriving to room) since the client has a diagnosis of meningococcal meningitis. Neurological checks can be completed after the client is placed in isolation. 2. Incorrect: Collecting data for the health history can be obtained after placing the client on droplet precautions. 4. Incorrect: Orientation can occur after the initiation of droplet precautions.

Which task would be appropriate for the LPN/VN to accept from the Labor, Delivery, Recovery, Postpartum (LDRP) charge nurse? You answered this question Correctly 1. Administer IV pain medication to a client three days postopertive cesarean section. 2. Draw a trough vancomycin level on a client 3 days postpartum with bilateral mastitis. 3. Reinforce how to perform perineal care to a primipara who is four hours postpartum. 4. Draw admission labs on a client admitted in final stages of labor.

3. Correct: Client teaching may be reinforced by an LPN/VN on a stable client. 1. Incorrect: Administering IV pain medications is out of the scope of practice of LPN/VN. 2. Incorrect: This client with a severe infection who is only 3 days postpartum is considered an unstable client. Therefore, this client should have the blood drawn and receive care from the RN. 4. Incorrect: Drawing admission labs on a client in the final stages of labor would be inappropriate, because the client is potentially unstable and needs experienced LDRP nursing care.

To determine the standards of care for the institution, the nurse should consult which document? You answered this question Correctly 1. Organizational Chart 2. Personnel policies 3. Policies and procedure manual 4. Job descriptions

3. Correct: The standard of care for nurses is defined as what a prudent nurse would do in the same situation. The policies and procedure manual defines standards of care for an institution. 1. Incorrect: An organizational chart identifies which departments exist and their relationship with each other. It also lists the chain of command of positions/jobs. 2. Incorrect: Policies for personnel are not standards of care. Personnel policies are pre-established guidelines that are utilized to govern personnel related issues. 4. Incorrect: A job description is a formal written statement of an employee's work responsibilities. This is not related to the standards of care and therefore, doesn't go into detail about standards of care.

The nurse is caring for a depressed client. The client has a flat affect, apathy, and slowed physical movement. The client has not bathed in several days and there is a malodorous odor noted. Which intervention would be most appropriate at this time? You answered this question Incorrectly 1. Explain the rules about daily showers. 2. Leave the client alone since there is slowed movement. 3. Tell the client it is time to take a shower. 4. Ask when he or she would like to take a shower.

3. Correct: Depressed clients often have little energy to do or think. The depression can lead to a lack of self-care as the client lacks motivation and energy. Give short, simple commands during this time. 1. Incorrect: Not very therapeutic. This is difficult for the depressed client to comprehend at this time. Give short, clear, simple commands. 2. Incorrect: Do not ignore the problem. You must do what is best for the client, and this would not be the best decision. Depression does include slowed movement, but being depressed is not a reason for the client to not meet hygiene needs. 4. Incorrect: The client will not want to do anything at this time. It will be put off, and depressed clients often have difficulty making decisions. The depressed client lacks motivation and energy. The depressed client needs short, simple statements.

A new admit arrives to the nursing unit with one thousand dollars in cash. What would be the best action by the nurse to safeguard the client's money? You answered this question Correctly 1. Insist the money go home with the client's visitor. 2. Place the money in the client's bedside table drawer. 3. Put itemized cash in envelope and place in hospital safe. 4. Lock money up in narcotic cabinet with client's identity and room number.

3. Correct: It is best to have two witnesses (preferably hospital staff) sign the inventory list. The best action by the nurse would be to itemize the valuables, place in an envelope with the record of the inventory, and have it put in the hospital safe. If you fail to properly safeguard the client's property, the trust of the healthcare team for medical care can also be lost. Liability waivers should be signed if for whatever reason, the valuables must remain at the hospital. 1. Incorrect: This is not the best option. The visitor may not be the best person to send the money with. The client also has the right to refuse. Sending the money home with someone else does not safeguard the client's valuables and puts the nursing unit at risk for liability if a liability waiver has not been signed. 2. Incorrect: This is not a safe option. Anyone could retrieve the money. This would be considered careless actions by the nurse and could cause a lack of trust in the entire healthcare team. 4. Incorrect: This is not a safe option. Anyone with access could retrieve the money. Although it is in a locked area, it does not need to be placed with narcotics where the cabinet would be accessed by multiple people. This would still be considered a failure to properly safeguard the client's valuables.

A nurse invites a friend home one evening. On arrival, the friend sees the nurse's large, white, long-haired cat sitting on the couch and begins to experience palpitations, trembling, nausea, shortness of breath, and a feeling of losing control. What should the nurse do first? You answered this question Incorrectly 1. Stay with the friend until the friend feels better. 2. Have the friend breathe into a paper bag. 3. Remove the cat from the room. 4. Dim the lights in the room.

3. Correct: Remove the source of the panic attack first, then continue to assess the person for symptoms. Removing the cat "fixes" the problem. 1. Incorrect: This is a correct answer, but remove the cat first. 2. Incorrect: This will help hyperventilation if it occurs, but the stem of the question said the client was having "shortness of breath" related to seeing the cat. Remove the cat first as this will fix the problem and alleviate the symptoms. 4. Incorrect: This is a correct answer, but remove the cat first.

The nurse is working with the parents of a preschooler to help promote healthy sleep patterns of approximately 8 - 12 hours of sleep per night. Which intervention should assist the parents to achieve adequate sleep for their preschooler? You answered this question Correctly 1. Offer a time of exercise prior to bedtime. 2. Follow a bedtime routine at least three or four nights per week. 3. Spend about 30 minutes with the preschooler prior to bedtime for stories. 4. Encourage preschooler to sleep without a toy.

3. Correct: Rituals help the preschooler to feel secure. Quiet time to read, tell stories, and say prayers prepares the child for sleep. 1. Incorrect: Stimulation of activity before bedtime impedes sleep. 2. Incorrect: The routine should be maintained each night if at all possible. Only through routine does the child feel secure in preparation for bedtime. 4. Incorrect: A special toy helps the child to feel secure and adds to the nighttime routine.

The nurse is caring for a client taking spironolactone. Which needed dietary change should the nurse reinforce to the client? You answered this question Correctly 1. Eat extra helpings of bananas. 2. Increase intake of water. 3. Avoid salt substitutes. 4. Increase intake of green leafy vegetables

3. Correct: Spironolactone is a potassium sparing diuretic. Salt substitutes have potassium instead of sodium and should be avoided. 1. Incorrect: Bananas have potassium and should be avoided to prevent hyperkalemia and life threatening arrhythmias. 2. Incorrect: Water intake does not affect the use of spironolactone and is not the best answer to the question. 4. Incorrect: Green leafy vegetables contain vitamin K and are not contraindicated. This is a distractor to the answer that could cause the most harm to the client if not chosen.

The nurse prepares a sterile field for a procedure. Fifteen minutes later, the nurse is informed that there will be a 20 minute delay before the primary healthcare provider will arrive. What action should the nurse take? You answered this question Incorrectly 1. Cover the sterile field with a sterile drape 2. Close and tape the doors so that no one may enter. 3. Monitor the sterile field while awaiting the primary healthcare provider. 4. Take down the sterile field

3. Correct: The nurse should monitor the sterile field while awaiting the primary healthcare provider. This means keeping the sterile field in your site. 1. Incorrect: Sterile fields should not be covered. Although there are no research studies to support or discount the practice, removing a table cover may result in a part of the cover that was below the table level being drawn above the table level or air currents drawing microorganisms from a nonsterile area to the sterile field. It is important to continuously monitor all sterile areas for possible contamination.​ 2. Incorrect: There is no specified amount of time designated that a room can remain open and not used and the sterile field still be considered sterile. The sterility of an open sterile field is event-related. An open sterile field requires continuous visual observation. Direct observation increases the likelihood of detecting a breach in sterility.​ 4. Incorrect: It is unnecessary to take down the sterile field as the delay is minimal. This is also an added cost to discard materials and redo the sterile field when it has not been contaminated. Sterile fields should be prepared as close as possible to the time of use. The potential for contamination increases with time because dust and other particles present in the ambient environment settle on horizontal surfaces over time. Particulate matter can be stirred up by movement of personnel when opening the room and can also settle on opened sterile supplies.

Two hours following a lumbar puncture, the client stands up to void and reports a headache rated 8 out of 10 on a pain scale. What priority action should the nurse perform? You answered this question Incorrectly 1. Instruct the client to drink at least 8 ounces (240 mL) of water. 2. Close room blinds to darken the environment. 3. Assist the client into a supine position in bed. 4. Notify primary healthcare provider of client's complaints.

3. Correct: The most frequent cause of headache following a lumbar puncture is loss of, or leaking, of cerebrospinal fluid from the puncture site. Positioning a client is an important nursing responsibility, particularly in this situation since the supine position could help to stop any leaking. Following this, the nurse will pursue additional actions as prescribed by the primary healthcare provider, which may include increasing fluids or even a blood patch. 1. Incorrect: Although increasing fluids may help clients under specific circumstances, it is not the priority action in this situation. Additionally, the primary healthcare provider may order IV fluids rather than PO fluids. 2. Incorrect: A darkened room can be beneficial for clients with severe migraine headaches, but would not be useful to this client. Headaches following a lumbar puncture are caused by the loss of cerebrospinal fluid and would not respond to a quiet, dark environment. So this would not be the priority. 4. Incorrect: Although the primary healthcare provider should indeed be notified of this situation, the nurse's priority action should first focus on stabilizing the client by addressing the cause of this problem and positioning the client.

What action should the nurse take after entering the room of a client who becomes agitated and combative? You answered this question Correctly 1. Administer prn sedative. 2. Notify the family of client behavior. 3. Speak softly to the client. 4. Apply wrist restraints

3. Correct: The nurse needs to present a calm manner and speak quietly to the client. This will build trust and decrease tension and stress in the client. 1. Incorrect: This is a form of restraint. The use of positive nursing actions can reduce the use of chemical (drug therapy) restraints. 2. Incorrect: Do not pick an answer that transfers the client away from the nurse's care. 4. Incorrect: Restraints are a last resort and can make the client more agitated.

After shift report, which client should the nurse see first? You answered this question Correctly 1. Eight year old that is in skeletal traction. 2. Six year old who is 5 hours postop appendectomy. 3. Unattended two year old admitted for a sleep study. 4. Four year old cerebral palsy child with a urinary tract infection (UTI).

3. Correct: The unattended child should be checked first to make sure he/she is safe and having no complications. A child this age is entirely dependent on someone else. Safety is priority here. 1. Incorrect: An eight year old in skeletal traction does not take priority over unattended 2 year old.. 2. Incorrect: A six year old who is 5 hr post appendectomy should be seen, but not as immediate as an unattended 2 year old. 4. Incorrect: This client has a UTI and is not acutely ill at this time. The major clues in the question are age and unattended. The unattended two year old child's safety is the reason for that child being the priority over this child.

The nurse is caring for a client who has taken an acetaminophen overdose. Which symptom is the client most likely to exhibit? You answered this question Correctly 1. Expectorating pink frothy sputum 2. Sudden onset of mid-sternal chest pain 3. Jaundiced conjunctiva 4. Diaphoresis and fever

3. Correct: This is a sign of liver damage, which is caused by an overdose of acetaminophen. 1. Incorrect: This is a symptom of pulmonary edema, not liver damage. 2. Incorrect: This is a symptom of myocardial ischemia, not liver damage. 4. Incorrect: Acetaminophen would decrease fever, and fever could cause diaphoresis, so neither of these are expected with acetaminophen overdose.

Which task would be most appropriate for the nurse to assign to an unlicensed assistive personnel (UAP)? You answered this question Incorrectly 1. Obtain a sterile urine specimen from an indwelling catheter. 2. Insert an in-and-out catheter on a postpartum client. 3. Take vital signs on a client 12 hours postpartum. 4. Remove an indwelling catheter on a postpartum client. 5. Perform perineal care on a client with an episiotomy.

3., & 5. Correct: Taking vital signs is within the scope of practice for the UAP, but the nurse is responsible for reviewing the vital signs. Performing perineal care is within the scope of practice for the UAP. The nurse is responsible for assessing the episiotomy and confirming that perineal care is done properly. 1. Incorrect: Invasive procedures are not appropriate tasks for UAP (obtaining sterile specimen from indwelling catheter). 2. Incorrect: Invasive procedures are not appropriate tasks for UAP (inserting catheter). 4. Incorrect: Invasive procedures are not appropriate tasks for UAP (removing indwelling catheter).

Which nursing intervention should the nurse include when caring for a client with Alzheimer's disease being admitted to a long term care facility? You answered this question Correctly 1. Offer multiple environmental stimuli at the same time to provide distraction. 2. Encourage the client to participate in activities such as board games. 3. Restrain the client in a chair to prevent falls when sundowning occurs. 4. Involve the client in supervised walking as a routine.

4. Correct: A regular routine and physical activity help clients with Alzheimer's disease maintain abilities for a longer period of time. Physical activities promote strength, agility, and balance. The client's walking should be supervised for client safety issues. 1. Incorrect: Environmental stimuli should be limited with clients with Alzheimer's disease. The client can become agitated and/or more disoriented with an increase in environmental stimuli. 2. Incorrect: Board games would not be appropriate due to the client's cognitive and memory impairment. Board games require complex cognitive actions. 3. Incorrect: Restraints should be avoided because they increase agitation. The client may become agitated by the restriction of the restraints. Also the client may perceive the restraints as a threat.

Which is a risk factor for developing breast cancer in women? You answered this question Correctly 1. Menopause before the age of 50 2. Drinking one glass of wine daily 3. Multiparity 4. Menarche at age 10

4. Correct: Early menarche, before age 12, is a known risk factor for breast cancer. The increased risk of breast cancer linked to a younger age at the first period is likely due, at least in part, to the amount of estrogen a woman is exposed to in her life. A higher lifetime exposure to estrogen is linked to an increase in breast cancer risk. The earlier a woman starts having periods, the longer her breast tissue is exposed to estrogens released during the menstrual cycle, and the greater her lifetime exposure to estrogen. 1. Incorrect: Studies show women who go through menopause after age 50 have increased risk of breast cancer. The risk for breast cancer increases as the time period between menarche and menopause increases. 2. Incorrect: There is a small increase in risk with moderate alcohol consumption, not one glass of wine daily. Drinking low to moderate amounts of alcohol, however, may lower the risks of heart disease, high blood pressure, and death. But, drinking more than one drink per day (for women) and more than two drinks per day (for men) has no health benefits and many serious health risks, including breast cancer. Alcohol can change the way a woman's body metabolizes estrogen (how estrogen works in the body). This can cause blood estrogen levels to rise. Estrogen levels are higher in women who drink alcohol than in non-drinkers. These higher estrogen levels may, in turn, increase the risk of breast cancer. 3. Incorrect: Nulliparity (no pregnancies) is a known risk factor for breast cancer. Factors that increase the number of menstrual cycles also increase the risk of breast cancer, probably due to increased endogenous estrogen exposure.

What should the nurse calculate as the estimated due date of a pregnant client, whose last menstrual period started on August 31st? You answered this question Correctly 1. July 1st 2. May 6th 3. May 31st 4. June 7th

4. Correct: Estimated date of delivery is calculated by counting back 3 months, adding seven days, and adding a year if needed. August 31st - 3 months would be May 31st. Add seven days. The estimated date of delivery would be June 7th. 1. Incorrect: Estimated date of delivery is calculated by counting back 3 months, adding seven days, and adding a year if needed. 2. Incorrect: Estimated date of delivery is calculated by counting back 3 months, adding seven days, and adding a year if needed. 3. Incorrect: Estimated date of delivery is calculated by counting back 3 months, adding seven days, and adding a year if needed.

The nurse is reinforcing teaching to a client who has been prescribed glucocorticoids for the treatment of Addison's disease. What points should the nurse emphasize? You answered this question Incorrectly 1. Be aware of the development of hypoglycemia. 2. Test the urine for albumin or other proteins. 3. Take the medication 30 minutes prior to bedtime. 4. Maintain the prescribed dose without interruption in therapy.

4. Correct: Glucocorticoids should not be abruptly discontinued due to the risk of adrenal suppression with prolonged use. Doses should be tapered before being discontinued. 1. Incorrect: Increased blood sugar is an adverse effect associated with glucocorticoid therapy, not hypoglycemia. 2. Incorrect: Protein in the urine is not associated with glucocorticoid therapy. 3. Incorrect: Insomnia is an adverse effect associated with glucocorticoid therapy. Daily dosing of glucocorticoids should be done in the morning to decrease this effect.

Which foods should the nurse encourage a client to avoid when prescribed a diet limiting purine rich foods? You answered this question Incorrectly 1. Peanut butter 2. Potatoes 3. Apples 4. Venison 5. Scallops

4., & 5. Correct: Purines are found in many foods and produced naturally by the body. Meats such as liver, bacon, veal, and venison are high in purine and should be avoided. Seafood such as sardines, mussels, codfish, scallops, trout, and haddock are high in purine and should be avoided. 1. Incorrect: Peanut butter is low in purine and allowed on a diet limiting purine rich foods. 2. Incorrect: Potatoes are allowed on low purine diet. 3. Incorrect: All fruit and fruit juices are low in purine. So apples can be eaten on a diet limiting purine rich foods.

The primary healthcare provider prescribes glycopyrrolate 0.2 mg IM thirty minutes prior to electroconvulsive therapy (ECT). What should be the nurse's response when the client asks why this drug is being given? You answered this question Correctly 1. "The action of the medication is complex." 2. "This drug will prevent you from having a seizure." 3. "This medication will relax your muscles so that you do not break a bone." 4. "Glycopyrrolate will decrease stomach secretions."

4. Correct: Glycopyrrolate is an anticholinergic. Glycopyrrolate blocks the activity of acetylcholine which reduces secretions in the mouth, throat, airway, and stomach. It is used prior to procedures to decrease the risk of aspiration. 1. Incorrect: The client has a right to be told the reason the drug is given. This is a nontherapeutic communication response. The nurse should not refuse the client's desire to understand their medications. 2. Incorrect: Glycopyrrolate blocks the secretions in the mouth, throat, airway and stomach. The medication does not prevent the client from having a seizure. The ECT will induce a seizure, which is the desired action. 3. Incorrect: This is not the drug's purpose, so this would be incorrect information to give to the client.

A client has an intestinal obstruction and a NG tube to low suction. Blood gases are ph 7.54, pCO2 40, HCO3 35. The client is weak, shaky, and reports tingling of the fingers. The nurse knows that this client is most likely in which acid base imbalance? You answered this question Correctly 1. Respiratory acidosis 2. Respiratory alkalosis 3. Metabolic acidosis 4. Metabolic alkalosis

4. Correct: Metabolic alkalosis happens when there is a loss of acid or a gain in bicarbonate. Metabolic alkalosis occurs from gastric losses via vomiting, NG tubes to suction, or lavage, and potent diuretics. Signs and symptoms include n/v, sensorium changes, tremors, and convulsions. pH > 7.45, pCO2 normal between 35-45, HCO3 > 26. 1. Incorrect: This is not a respiratory related acid-base imbalance 2. Incorrect: This is not a respiratory related acid-base imbalance 3. Incorrect: This is not acidosis. There is loss of gastric acid from the stomach due to the NG tube to low suction.

Which pain scale should the nurse use to monitor the pain level of a 3-year old client after surgery? You answered this question Correctly 1. Numerical scale 2. Verbal descriptive scale 3. Visual analog scale 4. FACES scale

4. Correct: Monitoring pain in children requires special techniques. The nurse should use the FACES scale as a tool to assess this client's pain level. Children as young as 3 years of age can use the FACES scale to communicate their pain level to the medical team. The scale has six faces ranging from smiling face to sad, tearful face. 1. Incorrect: Not age appropriate. This scale uses numbers. 2. Incorrect: Not age appropriate. Young children may not understand the word pain. 3. Incorrect: Not age appropriate. This scale requires reading.

Post cataract removal, a client reports nausea and severe pain in the operative eye. Which nursing intervention takes priority? You answered this question Correctly 1. Administer morphine and ondansetron. 2. Reposition client to non-operative side. 3. Massage the canthus to unblock the lacrimal duct. 4. Notify the primary healthcare provider.

4. Correct: Severe pain and nausea indicate an increase in intraocular pressure and must be reported at once. Eye damage can result if not resolved quickly. The primary healthcare provider may prescribe medications or take the client back to surgery. 1. Incorrect: This is not the priority, as severe pain and nausea indicate an increase in intraocular pressure and must be reported at once. 2. Incorrect: Repositioning will not fix the problem. Severe pain with nausea indicate an increase in intraocular pressure and must be reported at once. 3. Incorrect: The problem is an increase in intraocular pressure which needs to be reported to the primary healthcare provider.

The nurse is preparing to make an occupied bed. Which action by the nurse is important to preserve client's self-esteem during this procedure? You answered this question Correctly 1. Remove the top sheet first and replace with a clean one. 2. Inform the client that they will be uncovered only for a short time. 3. Ask the client to relax as the top sheet is removed and the bottom sheet is changed. 4. Cover the client with a bath blanket before removing any of the sheets on the bed.

4. Correct: The client should not be exposed during the bed change. Cover with a bath blanket as the top sheet is removed. 1. Incorrect: The client's self-esteem will not be preserved if uncovered during the procedure. Being exposed to the nurse is very troubling for most clients. 2. Incorrect: The client should be covered throughout the procedure. 3. Incorrect: The client's self-esteem will not be preserved by relaxing. Being exposed is anxiety provoking, and exposure is unnecessary.

Upon receiving a diagnosis of Stage 4 lung cancer, an elderly client expresses regret for having chosen to smoke. Which response by the nurse would best help the client cope at this time? You answered this question Correctly 1. "You are lucky to have lived a very long life." 2. "We have younger clients in worse shape than you." 3. "The doctor will make sure to treat any pain." 4. "You are regretting your decision to smoke."

4. Correct: The nurse responds with an open-ended statement that reflects back to what the client has stated. This allows the client to continue expressing concerns and feelings about the diagnosis or past choices. At this time, encouraging the client to verbalize is the best choice to help with coping. 1. Incorrect: This is a closed-ended statement that diminishes the client's feelings about the diagnosis or possible personal choices that may have led to this situation. The response infers that the client should be grateful for the life lived and belittles the client's response. 2. Incorrect: Such a non-therapeutic statement does not address the client's feelings and, in fact, devalues the client by comparing this situation to that of other clients. The nurse should focus on helping the client to cope at this time by encouraging the expression of feelings. 3. Incorrect: Rather than allowing the client to verbalize concerns or emotions, the nurse has responded with a close-ended statement that addresses a topic not initiated by the client's comments. This option does nothing to help the client cope, but rather may instill fear by referring to pain that may or may not occur.

The nurse monitors a multigravida who is four hours postpartum. Findings include that fundus is firm, 1 centimeter above the umbilicus, and deviated to the right side. The lochia is moderately heavy and bright red. Which nursing intervention has priority? You answered this question Correctly 1. Massage the fundus. 2. Administer intravenous oxytocin. 3. Document these normal findings. 4. Assist the client up to void.

4. Correct: These findings are caused by a full bladder, which prevents the uterus from contracting down and achieving homeostasis. Once the bladder is empty, the fundus will contract adequately and return to its normal location at level of umbilicus or 1 finger breadth below the umbilicus and in the midline. A distended bladder will displace the uterus, usually to the right. 1. Incorrect: The nurse may check fundus after client voids to ensure that this fixes the problem. 2. Incorrect: Administering oxytocin is not the first intervention for this issue. 3. Incorrect: These are not normal findings, so this would be incorrect information for the nurse to document.

The nurse is identifying home safety issues to prevent injury for a visually impaired elderly client who also has diabetes. Which findings are important for the nurse to include in this process? You answered this question Correctly 1. Episodes of mild anxiety 2. Rugs secured to the floor 3. Adequate lighting 4. Functional eye glasses 5. Client is wearing well-fitting closed toe shoes

Before we review the options, let's look at the question. The key words in a question should be identified. The key words in this question are home assessment, preventing injury, visually impaired elderly client, and diabetes. Each option stands alone with the question. So let's look at the options... Remember client safety is always a priority. 1. This is false because mild anxiety is not likely to result in injury. Symptoms of anxiety may include a feeling of overwhelming fear, trembling or a surge of doom and gloom. 2. Rugs that are secured to the floor can cause the client to trip or slip. True. The rugs should not move when stepped on or have elevated ends that may cause the client to trip. 3 During the night, falls can be reduced by having adequate lighting throughout the home. True. This client has been identified as a high risk for falls. The home fall prevention education program identifies practical interventions to reduce the possibility of the client falling. All obstacles such as furniture cannot be removed but, a night light will make the obstacles more visible at night. This will assist in reducing the client's possibility of falling over them. This is a safety intervention. 4. The visually impaired client needs functioning glasses to maximize sight and safety within the home. True. 5. The client should wear well-fitting closed toes shoes. True. Home fall prevention should emphasize that the client should always wear properly fitting shoes that have nonskid protection. The diabetic client should wear closed toe shoes to prevent injury to the toes. This safety intervention will decrease their risk for falls.

The nurse, caring for a client diagnosed with Alzheimer's disease (AD), notices the client becoming agitated. What nursing intervention would be appropriate for the nurse to initiate? You answered this question Incorrectly 1. Provide a snack for the client. 2. Tell the client to stop the unwanted behavior. 3. Take client for a walk. 4. Ask the client to sweep the floor. 5. Inform the client that restraints will be used if behavior continues. 6. Turn on the client's favorite music.

Behavioral problems occur in about 90% of clients diagnosed with Alzheimer's. These include repetitiveness, delusions, hallucinations, agitation, aggression, altered sleeping patterns, wandering, and resisting care. These behaviors are often unpredictable and may challenge caregivers. Caregivers need to be aware that these behaviors are not intentional. Nursing strategies that address difficult behavior include redirection, distraction, and reassurance. Option 1: Good choice. This nursing action will serve as a distraction and redirection. This changes the client's focus and helps decrease agitation. The AD client also may be malnourished and underweight and benefit from a healthy snack. Option 2: No. The client with AD may not be able to control their behavior. This is a behavioral change that comes with the disease. Option 3: Yes! Taking the client for a walk involves exercise and change of environment. This serves as redirection and distraction. Option 4: Another good choice. Redirection and distraction are helpful in controlling agitation. Sweeping the floor gives the AD client a purposeful activity. Option 5: Oh no - wrong! Stay away from restraints! This is a very last resort. Also, this is threatening the client and would most likely increase agitation. Reassurance involves communicating to the client that he or she will be protected from harm, danger, and embarrassment. Reassurance is an appropriate nursing strategy for AD. Option 6: Yes! Use of songs, poems, music, massage, and favorite objects can soothe the client.

Place the steps in order that the nurse should take to administer a subcutaneous injection. You answered this question Correctly

First perform hand hygiene. Then apply gloves and locate injection site using anatomical landmarks. Start at the center of the site and rotate outward in a circular direction to cleanse the site. Remove the needle cap by pulling the cap straight off. Next, hold the syringe and pinch the skin with nondominant hand. Inject the needle quickly then administer the medication slowly. Finally, dispose of the syringe in the sharps container

The nurse is caring for a client hospitalized with dissociative amnesia. Which nursing interventions are appropriate for this client? You answered this question Correctly 1. Obtain client likes and dislikes from family members. 2. Expose the client with data regarding the forgotten past. 3. Expose client to stimuli that was a happy memory of the past. 4. Hypnotize the client to help restoration of memory. 5. Ensure client safety.

First, think safety. Then, think of nursing interventions that will help relieve symptoms. Option 1: Do it. Obtaining information including client likes and dislikes will help the nurse plan client care. Also, this may help the client begin to remember and process the painful memories. Then the client can develop new coping skills. Option 2: No. The amnesia is protecting the client from memories that are traumatic. The memories can resurface after being triggered. Nursing interventions must focus on helping the client express and process painful memories in appropriate ways. This is not the way. Option 3: Yes! You want the client to remember happy times. This is therapeutic! Option 4: Oh no! The stem asks for a nursing intervention. Do not choose this option. Hypnosis may be a treatment for dissociative amnesia but is not a nursing intervention. Read the stem carefully. Option 5: Excellent. A priority is client safety. This disorder may be so distressing and cause behavior problems and confusion. An appropriate nursing intervention is client safety.

The clinic nurse answers a call from a client who is voicing intent to commit suicide. The client tells the nurse, "I am sitting here with a bottle of pain killers in my hand." What response by the nurse is appropriate? You answered this question Correctly 1. "I want to help you to resolve the problem." 2. "You should drive yourself to the emergency room." 3. "You did the right thing by calling." 4. "I want you to stay on the phone with me." 5. "Someone is calling 911 for an ambulance."

In this select all that apply question, you should read each option as a true or false statement. Remember to take all suicide plans by the client seriously. Keep the client safe! Option 1: Great choice! The nurse offers to help. By calling the clinic, the client is reaching out for help to prevent carrying out the suicide plan. Option 2: No way! This client is in great emotional pain and does not need to drive to the emergency department. This is unsafe! Option 3: Yes. The nurse is supporting the client in their call for help. This statement is supportive of the client. Option 4: For sure! Keeping the client on the phone will keep the client from carrying out the suicide plan. This allows time for emergency help to arrive. This is a very important nurse response. Option 5: A must! Both the 911 call and emergency personnel are needed. This client has definite suicide plans and the means (bottle of pain medication) to carry out the suicide plan.

What information should be reinforced for parents regarding the promotion of adequate bowel elimination in their toddler? You answered this question Incorrectly 1. Include adequate fiber in the diet through whole grains and fruits. 2. Increase intake of water daily. 3. Provide toileting opportunities that are free from distractions. 4. Encourage the toddler to go to the bathroom at least three times daily. 5. Take away attention from the toddler unable to potty.

Options 1 and 2 are true. Increasing fiber and fluid in the diet will promote adequate bowel elimination by increasing peristalsis. Option 3 is true. Toddlers who are distracted may not wait for complete bowel elimination before getting up from the toilet. Option 4 is false. The best time to have a bowel movement is after a meal when peristalsis increases and prior to bedtime. Therefore, the child is not simply encouraged to go to the bathroom, but should be taken at these times to promote elimination. Option 5 is false. A client's emotional status must be maintained. You do not want to punish the child if unable to have a bowel movement.

The primary healthcare provider prescribed tolbutamide 250 mg orally twice a day. The pharmacy dispensed tolbutamide 0.5 g scored tablets. How many tablets will the nurse administer? Round your answer using one decimal point. You answered this question Correctly Enter the answer for the question below.

Step 1: 1000 mg : 1 g = x mg: 0.5 g x = 500 mg Step 2: 500 mg: 1 tab = 250 mg : x tab 500 x = 250 X = 0.5

Which interventions are appropriate for the nurse to identify for a client admitted to the psychiatric unit for management of anorexia nervosa? You answered this question Incorrectly 1. Weigh daily at the same time each day. 2. Allow only 20 minutes of exercise daily. 3. Allow the client to bargain for privileges as long as the client eats. 4. Stay with the client during the established time for meals. 5. Maintain visual observation for 1 hour following meals.

Think nursing interventions. And how to manage anorexia nervosa. Option 1: A must! Weight loss programs recommend weighing weekly. But wait! This client with anorexia nervosa is seriously underweight. Weight gain is a life saving measure. The nurse needs to get a daily weight on this client in the psychiatric unit. And remember, if hospitalized, the anorexia nervosa is severe and causing acute symptoms in the client. Option 2: No. This inpatient client may be on restricted activity! A very individualized exercise plan prescribed by the primary healthcare provider is needed as the client improves. Option 3: No way. Bargaining is a nontherapeutic technique. The client will learn acceptable and unacceptable behaviors and how to obtain privileges. Option 4: Yes! Much can be observed by the nurse during meal time. The nurse can intervene appropriately during this time with positive reinforcement. Option 5: Oh yes! The anorexic client may engage in self-induced vomiting, although restricting intake is classic with anorexia nervosa. The client may try to hide food during meal time and may use the hour after meals to induce vomiting or discard the food due to client's fear of weight gain.

A client with cancer refuses treatment and asks about options for hospice home care. The client's daughter asks the nurse to talk the client into agreeing to cancer treatment. The nurse explains to the daughter that this violates which client right? You answered this question Incorrectly 1. Advocacy 2. Ability to decline participation in experimental treatments. 3. Expectation of reasonable continuity of care. 4. To make decisions about the plan of care

Under the Patient Self-Determination Act (PSDA), healthcare institutions provide clients with a summary of their rights when making health care decisions as well as the facility's policies regarding recognition of advanced directives. The client is advised of the right to consent to or refuse treatment or Self-determination. Client rights refer to such matters as access to care, dignity, confidentiality, and consent to treatment. The competent adult client has the right to participate in the plan of care, to refuse a proposed treatment, and to accept alternative care and treatment. Documentation should be made that the client fully understands the risks and benefits of their decision. Option 1. False: The right to advocacy relates to the right to have another person present during interviews or examinations. This right would not be violated by the nurse if attempts were made to convince the client to have cancer treatment. Option 2. False. The right to decline participation in research or experimental studies is incorrect because no research or experimental treatment is proposed to the client. Option 3. False. The right to expect reasonable continuity of care appears to be a possible correct answer, but is incorrect because the client has not been transferred to hospice home care. Attempting to convince the client to agree to cancer treatment would not be pertinent to continuity of care in this situation. Option 4. True. This is what self-determination is: the right to make decisions about one's own care.


Conjuntos de estudio relacionados

introduction to the primates &prosimians, tarsier

View Set

Chapter 57 Management of Patients with Burn injury

View Set

Life-span Developmental Psych Final (Santrock)

View Set

Iowa life insurance only course pt.2

View Set

yockers - unit 2 ap classroom quizes

View Set

EAQ 14: Inflammatory Intestinal Disorders

View Set

Chapter 3 homework (study guide)

View Set

Legal and Ethical Responsibilities Jted MA Practice Final

View Set